1 Introduction and Statement of Main Results

In this work, we study the following quasilinear elliptic problem involving a weighted 1-Laplacian operator:

$$\begin{aligned} \left\{ \begin{array}{ccc} -\Delta _{1}\left( \textrm{au} \right) +b\frac{u}{|u|}=\frac{h}{u^{1-q}} &{} \text{ in }\ {\mathbb {R}}^{N} ,\ N\ge 2, \\ u>0, &{} \text{ a.e. } \text{ in }\ {\mathbb {R}}^{N}, \end{array} \right. \end{aligned}$$
(1.1)

where \(\Delta _{1}\left( \textrm{au} \right) \) is the formal limit of the weighted p-Laplacian operator

$$\begin{aligned} \Delta _p (\textrm{a u}) = \text{ div }\left( a \left| \nabla u\right| ^{p-2} \nabla u\right) \end{aligned}$$

as \(p \rightarrow 1^+, \) i.e., \( \Delta _{1}\left( \textrm{au} \right) =\text{ div }\left( a \frac{{\mathrm{{Du}}}}{|{\mathrm{{Du}}}|} \right) .\)

The problem (1.1) is taken under the following assumptions:

\((H_{1})\): \(a \in L^1({\mathbb {R}}^N)\cap L^{\infty }({\mathbb {R}}^N)\cap C^1({\mathbb {R}}^N)\) such that

$$\begin{aligned} 0<a_R= \inf _{|x|\le R}a(x)<\infty ,\ \forall R>0. \end{aligned}$$

We also assume that that \( b \in L^1({\mathbb {R}}^N)\cap L^\infty ({\mathbb {R}}^N)\) and there exists a constant \( C_0 > 0 \) such that \( a(x) \le C_0 b(x),\ \forall \ x \in {\mathbb {R}}^N. \) Thus,

$$\begin{aligned} 0< b_R= \inf _{|x|\le R}b(x)<\infty ,\ \forall R>0. \end{aligned}$$

\((H_{2})\): \(h : {\mathbb {R}}^N \rightarrow {\mathbb {R}}\) is a nonnegative function such that \(\frac{h}{b}\in L^{\infty }({\mathbb {R}}^N)\) and

$$\begin{aligned} \int _{{\mathbb {R}}^N}\left( \frac{h}{b^r}\right) ^{\frac{1}{1-r}}\textrm{d}x<\infty ,\ \forall \ 0<r<1. \end{aligned}$$

Example: An example of functions ab and h satisfying the hypotheses \( (H_1) \) and \( (H_2) \) is given by:

$$\begin{aligned} a(x) = e^{- \alpha \left| x\right| },\ b(x) = e^{- \beta \left| x\right| },\ h(x) = e^{- \gamma \left| x\right| },\ 0< \beta < \min (\alpha , \gamma ). \end{aligned}$$

In the last years, problems involving the 1-Laplacian operator, formally defined by

$$\begin{aligned} \Delta _{1}u:=\text{ div }\left( \frac{{\mathrm{{Du}}}}{|{\mathrm{{Du}}}|}\right) , \end{aligned}$$

caught the attention of so many specialists in partial differential equations, mainly because of its applications in image processing and in fracture mechanics. Concerning this applied aspect of such a problems, one can, for example, cite [8] where the authors showed the applications of minimizing functionals with linear growth arising from the 1-Laplacian operator in image restoration. In [25], the authors studied the inverse mean curvature flow problem, by studying problems involving the 1-Laplacian operator and containing some gradient terms.

By studying problems involving the 1-Laplacian operator in some domain \( \Omega \) of \( {\mathbb {R}}^N,\ N \ge 2, \) we could think that the natural space to solve them is \(X=W^{1,1}(\Omega );\) however, it lacks the lower semi-continuity property of the norm \(W^{1,1}(\Omega )\) with respect to the topology of \(L^1(\Omega ).\) This, in turn, makes it extremely difficult to prove the existence of solution to equations involving the 1-Laplacian operator. Nevertheless, there is another Banach space, which contains \(W^{1,1}(\Omega ),\) where the proper extension of the norm is lower semi-continuous in some sense. This space is the so-called space of bounded variation functions BV\((\Omega ).\) It is well known that this space is not easy to manipulate because its dual is not well known, and by consequence this space is not necessarily reflexive, making difficult to prove compactness conditions like Palais–Smale and Cerami conditions.

There are plenty of works dealing with this highly singular operator by using different approaches. Among them, we can, for example, cite [18,19,20,21, 39] , which applied the approach based on the variational method using nonsmooth critical point theory for energy functionals to get a solution, where by a solution is understood a function in the space of functions of bounded variation, in which the sub-differential of the energy functional vanishes. Problems involving such an operator can also be addressed using another method as it will be the case in the present work, that is the approximation technique. This technique is mainly based on the idea of working with a class of p-Laplacian problem and then taking the limit when \(p\rightarrow 1^+\) to finally obtain a solution to the 1-Laplacian problem (see, for instance, [1, 9, 22, 24, 29,30,31,32,33, 35,36,37]). By using this approach, the concept of solution to equations involving the 1-Laplacian was developed by F. Andreu et al. (see [1] and the book [2] ) and F. Demengel (see [15]). Indeed, in [1], the authors characterize the imprecise quotient \(\frac{\mathrm{{Du}}}{|\mathrm{{Du}}|}\) (where \(|\textrm{Du}|\) denotes the total variation of the Radon measure Du), by Anzellotti’s pairing theory (see [3] and also [2]). This theory allows them to introduce a vector field \(z\in L^\infty (\Omega , {\mathbb {R}}^N)\) such that \(\left( z, \mathrm{{Du}}\right) = \left| \textrm{Du}\right| \) in the sense of measures, what means that z somehow plays the role of \(\frac{\mathrm{{Du}}}{|\mathrm{{Du}}|},\) where \( (z, \textrm{Du}) \) is a Radon measure appropriately defined in such way that it can be considered as replacing the dot product in \( {\mathbb {R}}^N. \)

Recently, V. De Cicco, D. Giachetti and S. Segura de León in [14] used the approximation technique to deal with a problem similar to (1.1). More precisely, V. De Cicco, D. Giachetti and S. Segura de León studied the following Dirichlet problem for an equation involving the 1-Laplacian operator and a singular lower-order term

$$\begin{aligned} {\left\{ \begin{array}{ll} -\text{ div }\left( \frac{\mathrm{{Du}}}{|\mathrm{{Du}}|}\right) =&{} \frac{f(x)}{u^{\gamma }}\ \text{ in }\ \Omega ,\\ u=0, &{} \text{ on }~~\partial \Omega , \end{array}\right. } \end{aligned}$$
(1.2)

where \(\Omega \subset {\mathbb {R}}^N\) is a bounded open set with Lipschitz boundary \(\partial \Omega ,\) \(0< \gamma <1\) and f is a function belonging to \(L^N(\Omega ).\) In that work, through an approximation scheme, they proved the existence of a BV-solution. The boundedness of the domain played a crucial role to prove some a priori estimates and also in the process of the passage to the limit. A similar problem has been treated in [13]. In [29], the authors dealt with the case when the source f belongs to \( L^1( {\mathbb {R}}^N). \) Finally, we have also to cite a recently published article [17] where the authors investigated an equation similar to (1.2) but containing a gradient term.

In the present work, we firstly deal with the following approximated singular p-Laplacian equation related to (1.1):

$$\begin{aligned} \left\{ \begin{array}{ll} -\Delta _p \left( \textrm{a u} \right) +b|u|^{p-2}u=\frac{h}{u^{1-q}} &{}\quad \text{ in }\ {\mathbb {R}}^{N} ,\ N\ge 2, \\ u>0, &{}\quad \text{ in }\ {\mathbb {R}}^{N}. \end{array} \right. \end{aligned}$$
(1.3)

The existence of a solution \( u_p \) to (1.3) is proved using variational tools. In order to overcome the difficulties due to the existence of the singularity as well as the unboundedness of the domain, some new sophisticated arguments have to be employed. Some a priori estimates are proved. Next, we study the behavior of \(u_p\) as \(p\rightarrow 1^+,\) proving that,

$$\begin{aligned} u_p \rightarrow u\ \text{ pointwise } \text{ in }\ {\mathbb {R}}^N, \end{aligned}$$

and finally, we prove that this function u is a solution to our initial problem (1.1). Due to the fact that the weights a and b are unbounded from below, many difficulties arise when we try to establish the a priori estimates for the family \( ( u_p)_{1< p < 2} \) and also when passing to the limit as \( p \rightarrow 1^+. \)

Before stating our main results, let us define the space in which we study the problem (1.1). Since in (1.1) we are dealing with the weighted 1-Laplacian operator, the standard BV\(( {\mathbb {R}}^N)\) space cannot be used to analyze this problem. In fact, we will consider a weighted space, denoted by \(\textrm{BV}^b_{a}({\mathbb {R}}^N), \) used as functional framework. Such kind of weighted space has been introduced in [5] and used later by J.C. Ortizo Chata, M.T.O. Pimenta and S.S. Léon in [6] to treat the following elliptic equation containing a unbounded weights:

$$\begin{aligned} \left\{ \begin{array}{ll} - \text{ div }\left( \frac{1}{\left| x\right| ^a} \frac{\mathrm{{Du}}}{\left| \mathrm{{Du}}\right| }\right) = \frac{1}{\left| x\right| ^b} f(u),&{}\quad \text{ in }\ \Omega , \\ u = 0,&{}\quad \text{ on }\ \partial \Omega , \end{array} \right. \end{aligned}$$

where \( \Omega \) is a bounded regular open set of \( {\mathbb {R}}^N,\ N \ge 2, \) containing the origin and the parameters a and b satisfy \( 0< a < N-1 \) and \( 0< b < a+1, \) and f is some continuous function with polynomial behavior satisfying some appropriate conditions. The weighted space \(\textrm{BV}^{b}_{a}({\mathbb {R}}^{N^{}})\) is the space of functions \(u:{\mathbb {R}}^N \rightarrow {\mathbb {R}}\) measurable such that \(b(x)u\in L^1({\mathbb {R}}^N)\) and a(x)Du is a finite Radon measure, i.e.,

$$\begin{aligned} \textrm{BV}^{b}_{a}({\mathbb {R}}^N)=\Big \{u:{\mathbb {R}}^N \rightarrow {\mathbb {R}}~~ \text{ measurable };\ bu\in L^1({\mathbb {R}}^N),\ a\mathrm{{Du}}\in {\mathcal {M}}( {\mathbb {R}}^N,{\mathbb {R}}^N) \Big \}, \end{aligned}$$

where \( {\mathcal {M}}( {\mathbb {R}}^N, {\mathbb {R}}^N) \) is the set of finite vectorial Radon measures.

See Sect. 2 for more details concerning the space \( \textrm{BV}_a^b( {\mathbb {R}}^N). \)

Definition 1

We say that \(u\in \textrm{BV}^{b}_{a}({\mathbb {R}}^N)\cap L^{\infty }({\mathbb {R}}^N) \) is a weak solution to problem (1.1) if \( u(x) \ge 0 \) a.e. \( x \in {\mathbb {R}}^N \) and there exist \(z\in {\mathcal {D}}{\mathcal {M}}^{\infty }_{a, \textrm{loc}}({\mathbb {R}}^N)\) with \(|z|_{L^{\infty }( {\mathbb {R}}^N, {\mathbb {R}}^N)}\le 1\) and \(\gamma \in L^ \infty ({\mathbb {R}}^N)\) with \(| \gamma |_{L^{\infty }( {\mathbb {R}}^N)}\le 1\) such that

$$\begin{aligned} {\left\{ \begin{array}{ll} \frac{h}{u^{1-q}}\in L^1_{\textrm{loc}}({\mathbb {R}}^N),\\ \chi _{\{u>0\}}\in \textrm{BV}_{\textrm{loc}}({\mathbb {R}}^N),\\ -\text{ div }\big (\textrm{az}\big )\chi ^*_{\{u>0\}} +b\gamma \chi _{\{u>0\}}=\frac{h}{u^{1-q}} \chi _{\{u>0\}},\ \text{ in }\ D'({\mathbb {R}}^N),\\ \big (\textrm{az},\mathrm{{Du}}\big )= a|\mathrm{{Du}}|\ \text{ as } \text{ measures } \text{ in }\ {\mathbb {R}}^N\ \text{ and }\ \gamma u=|u| \ a.e.\text{ in } \ {\mathbb {R}}^N \end{array}\right. } \end{aligned}$$
(1.4)

where \(\chi _{\{u>0\}}\) is the characteristic function of the region \({\{u>0\}},\ \chi ^*_{\{u>0\}} \) its precise representative,

$$\begin{aligned} {\mathcal {D}}{\mathcal {M}}^{\infty }_{a, \textrm{loc}}({\mathbb {R}}^N)=\{ z\in L^{\infty }({\mathbb {R}}^N, {\mathbb {R}}^N);\ \text{ div }\left( \textrm{az}\right) \in {\mathcal {M}}_{\textrm{loc}}({\mathbb {R}}^N)\}, \end{aligned}$$

\({\mathcal {M}}_{\textrm{loc}}({\mathbb {R}}^N)\) is the space of Radon measures which are locally finite in \({\mathbb {R}}^N,\) and the pairing \( (\textrm{az}, \mathrm{{Du}}) \) is defined in Sect. 2.2 (it can be seen as replacing the dot product in \( {\mathbb {R}}^N). \)

Theorem 1.1

Under the assumptions \( (H_1) \) and \( (H_2), \) there exists a solution to the problem (1.1) in the sense of Definition 1.

Theorem 1.2

Suppose that \((H_1)\) and \((H_2)\) hold. We also assume that \(h(x)> 0, \) a.e. \( x \in {\mathbb {R}}^N. \) Then, the problem (1.1) has a solution \(u\in \textrm{BV}^{b}_{a}({\mathbb {R}}^N)\cap L^{\infty }({\mathbb {R}}^N) \) such that:

$$\begin{aligned} {\left\{ \begin{array}{ll} u(x)>0\ \text{ a.e. }\ x \in {\mathbb {R}}^N,\ \frac{h}{u^{1-q}}\in L^1({\mathbb {R}}^N),\\ \exists z\in L^\infty ({\mathbb {R}}^N,{\mathbb {R}}^N),|z|_{L^{\infty }( {\mathbb {R}}^N, {\mathbb {R}}^N)}\le 1,\ \text{ div }\left( \textrm{az}\right) \in L^1({\mathbb {R}}^N),\\ \big (\textrm{az},\mathrm{{Du}}\big )= a|\mathrm{{Du}}|\ \text{ as } \text{ measures } \text{ in }\ {\mathbb {R}}^N,\\ -\text{ div }\big (\textrm{az}\big ) +b=\frac{h}{u^{1-q}},\ \text{ in }\ D'({\mathbb {R}}^N). \end{array}\right. } \end{aligned}$$
(1.5)

Moreover, u is unique.

This paper is organized as follows: in Sect. 2 we study the functional space \(W^{1,p}_{a,b}({\mathbb {R}}^{N})\) (the functional framework in which we will study the problem (1.3)). We will also define the weighted space \(\textrm{BV}^{b}_{a}({\mathbb {R}}^N)\) and try to establish some of its important properties. Finally, we extend the Anzellotti pairing theory to include unbounded vector fields. In Sect. 3, we prove the existence of a nontrivial solution \(u_p\) to equation (1.3) and prove some a priori estimates for the family \( (u_p)_{1< p < 2}. \) To finish, in Sect. 4 we will proceed to the passage to the limit as p tends to \( 1^+ \) and by consequence show that the limit of \(u_p\) is actually a solution to our problem (1.1). More precisely, we will prove Theorems (1.1) and (1.2).

2 Preliminaries

In this section, we provide some fundamental properties of the functional spaces \(\textrm{BV}^{b}_{a}({\mathbb {R}}^{N})\) and \( W^{1,p}_{a,b}( {\mathbb {R}}^N) \) which will be used in the proof of Theorems 1.1 and 1.2. A weighted version of the theory of \(L^{\infty }-\)divergence−measure vector fields of Anzellotti’s type is established.

2.1 The Space \(\textrm{BV}^{b}_{a}({\mathbb {R}}^N)\)

In this subsection, we provide the most important properties of our functional space \( \textrm{BV}_a^b( {\mathbb {R}}^N). \) First of all, for \( u \in \textrm{BV}_\textrm{loc}( {\mathbb {R}}^N) \) and \( 1 \le j \le N, \) one can define the application

$$\begin{aligned} \left\langle a \frac{\partial u}{\partial x_j}, \varphi \right\rangle = \left\langle \frac{\partial u}{\partial x_j}, a \varphi \right\rangle ,\ \forall \ \varphi \in C_c( {\mathbb {R}}^N). \end{aligned}$$

Hence, we say that \( a \mathrm{{Du}} \in {\mathcal {M}}( {\mathbb {R}}^N, {\mathbb {R}}^N) \) if and only if, for all \( 1 \le j \le N, \) there exists a constant \( c_j > 0 \) such that

$$\begin{aligned} \left| \left\langle a \frac{\partial u}{\partial x_j}, \varphi \right\rangle \right| = \left| \left\langle \frac{\partial u}{\partial x_j}, a \varphi \right\rangle \right| \le c_j \left| \varphi \right| _{\infty },\ \forall \ \varphi \in C_c ({\mathbb {R}}^N). \end{aligned}$$

Theorem 2.1

The following statements are equivalent:

  • \(u\in \textrm{BV}^{b}_{a}({\mathbb {R}}^N);\)

  • \(bu\in L^1({\mathbb {R}}^N)\) and

    $$\begin{aligned} |a\mathrm{{Du}}|({\mathbb {R}}^N)= \sup \left\{ \int _{{\mathbb {R}}^N}u\ \text{ div }\ \varphi ,\ \varphi \in C^{1}_c({\mathbb {R}}^N, {\mathbb {R}}^N),\ \left| \varphi (x)\right| \le a(x),\ \forall x\in {\mathbb {R}}^N \right\} < \infty . \end{aligned}$$

Proof

By the virtue of the Riesz representation theorem for Radon measures, we know that there exists a real regular Borel measure \( \mu =(\mu _1, \mu _2,\cdots ,\mu _N) \) such that

$$\begin{aligned} \left\langle a \mathrm{{Du}}, \varphi \right\rangle= & {} \sum _{i=1}^{N}\left\langle a \frac{\partial u}{\partial x_i}, \varphi _i\right\rangle =\sum _{i=1}^{N}\int _{{\mathbb {R}}^N} \varphi _i d \mu _i=\int _{{\mathbb {R}}^N} \varphi d \mu ,\\ \forall \ \varphi= & {} (\varphi _1, \varphi _2,\cdot \cdot \cdot , \varphi _N) \in C_c( {\mathbb {R}}^N, {\mathbb {R}}^N). \end{aligned}$$

Thus, \( a\mathrm{{Du}} = \mu \) in the distributional sense. Moreover,

$$\begin{aligned} \begin{aligned} \left\| a \mathrm{{Du}}\right\|&= \sup \left\{ \left\langle a\mathrm{{Du}}, \varphi \right\rangle ,\ \varphi \in C_c( {\mathbb {R}}^N, {\mathbb {R}}^N),\ \left| \varphi \right| _{\infty } \le 1 \right\} \\&= \sup \left\{ \int _{{\mathbb {R}}^N} \varphi d \mu ,\ \varphi \in C_c( {\mathbb {R}}^N, {\mathbb {R}}^N),\ \left| \varphi \right| _{\infty } \le 1\right\} \\&= \left| \mu \right| ( {\mathbb {R}}^N), \end{aligned} \end{aligned}$$

where \(\left| \mu \right| ( {\mathbb {R}}^N)\) denotes the total variation of the Borel measure \( \mu . \)

From the density of \( C_c^{1}( {\mathbb {R}}^N,{\mathbb {R}}^N) \) in \( C_c( {\mathbb {R}}^N, {\mathbb {R}}^N),\) it follows

$$\begin{aligned} \begin{aligned}&\sup \left\{ \int _{{\mathbb {R}}^N} u\ \text{ div }( \varphi ) \textrm{d}x,\ \varphi \in C^{1}_c({\mathbb {R}}^N, {\mathbb {R}}^N),\ \left| \varphi (x) \right| \le a(x),\ \forall x\in {\mathbb {R}}^N \right\} \\&\quad =\sup \left\{ \int _{{\mathbb {R}}^N} u\ \text{ div }( a \varphi ) \textrm{d}x,\ \varphi \in C^{1}_c({\mathbb {R}}^N, {\mathbb {R}}^N),\ \left| \varphi (x) \right| \le 1,\ \forall x\in {\mathbb {R}}^N \right\} \\&\quad =\sup \left\{ \int _{{\mathbb {R}}^N} u\ \sum _{i=1}^{N}\frac{\partial }{\partial x_i} (a \varphi _i) \textrm{d}x,\ \varphi = (\varphi _1,\cdots ,\varphi _N) \in C^{1}_c({\mathbb {R}}^N, {\mathbb {R}}^N),\ \left| \varphi (x) \right| \le 1,\ \forall x\in {\mathbb {R}}^N \right\} \\&\quad =\sup \left\{ \sum _{i=1}^{N} \int _{{\mathbb {R}}^N} u \frac{\partial }{\partial x_i} (a \varphi _i) \textrm{d}x,\ \varphi = (\varphi _1,\cdots ,\varphi _N) \in C^{1}_c({\mathbb {R}}^N, {\mathbb {R}}^N),\ \left| \varphi (x) \right| \le 1,\ \forall x\in {\mathbb {R}}^N \right\} \\&\quad =\sup \left\{ -\sum _{i=1}^{N} \left\langle \frac{\partial u}{\partial x_i} ,a \varphi _i \right\rangle ,\ \varphi = (\varphi _1,\cdots ,\varphi _N) \in C^{1}_c({\mathbb {R}}^N, {\mathbb {R}}^N),\ \left| \varphi (x) \right| \le 1,\ \forall x\in {\mathbb {R}}^N \right\} \\&\quad =\sup \left\{ -\sum _{i=1}^{N} \left\langle a\frac{\partial u}{\partial x_i} ,\varphi _i \right\rangle ,\ \varphi = (\varphi _1,\cdots ,\varphi _N) \in C^{1}_c({\mathbb {R}}^N, {\mathbb {R}}^N),\ \left| \varphi (x) \right| \le 1,\ \forall x\in {\mathbb {R}}^N \right\} \\&\quad = \sup \left\{ \left\langle - a \mathrm{{Du}}, \varphi \right\rangle ,\ \varphi \in C^{1}_c({\mathbb {R}}^N, {\mathbb {R}}^N),\ \left| \varphi (x) \right| \le 1,\ \forall x\in {\mathbb {R}}^N \right\} \\&\quad = \sup \left\{ \int _{{\mathbb {R}}^N} \varphi d \mu ,\ \varphi \in C^{1}_c({\mathbb {R}}^N, {\mathbb {R}}^N),\ \left| \varphi (x) \right| \le 1,\ \forall x\in {\mathbb {R}}^N \right\} \\&\quad = \sup \left\{ \int _{{\mathbb {R}}^N} \varphi d \mu ,\ \varphi \in C_c({\mathbb {R}}^N, {\mathbb {R}}^N),\ \left| \varphi (x) \right| \le 1,\ \forall x\in {\mathbb {R}}^N \right\} \\&\quad =\left| \mu \right| ({\mathbb {R}}^N)=|a\mathrm{{Du}}|({\mathbb {R}}^N) = \left\| a \mathrm{{Du}}\right\| . \end{aligned} \end{aligned}$$

If we denote by \( \left| \mu \right| = \left| a \mathrm{{Du}}\right| = a(x) \left| \mathrm{{Du}}\right| , \) i.e.

$$\begin{aligned} \int _{B} a(x)\left| \mathrm{{Du}}\right| =|a \mathrm{{Du}}|(B) = \left| \mu \right| ( B),\ \forall \ \text{ Borel } \text{ set }\ B \subset {\mathbb {R}}^N, \end{aligned}$$

then we can see that \(u\in \textrm{BV}^{b}_{a}({\mathbb {R}}^N)\) if and only if \( bu\in L^1({\mathbb {R}}^N)\) and

$$\begin{aligned} \int _{{\mathbb {R}}^N} a\left| \mathrm{{Du}}\right| =\sup \left\{ \int _{{\mathbb {R}}^N} u\ \text{ div }( \psi ) \textrm{d}x,\ \psi \in C^{1}_c({\mathbb {R}}^N, {\mathbb {R}}^N),\ \left| \psi (x) \right| \le a(x),\ \forall x\in {\mathbb {R}}^N \right\} <+\infty . \end{aligned}$$

\(\square \)

Naturally, we endow the space \(\textrm{BV}_a^b({\mathbb {R}}^N)\) with the norm

$$\begin{aligned} \Vert u\Vert =\int _{{\mathbb {R}}^N} b|u| \textrm{d}x+\int _{{\mathbb {R}}^N} a\left| \mathrm{{Du}}\right| . \end{aligned}$$

Equipped with that norm, \( \textrm{BV}_a^b( {\mathbb {R}}^N) \) is a Banach space but it is not necessarily reflexive. The following semi-continuity property is fundamental.

Lemma 1

Let \( (u_n)_{n\ge 0} \subset \textrm{BV}_{a}^{b}( {\mathbb {R}}^N) \) and \( u \in L^1_{\textrm{loc}}({\mathbb {R}}^N) \) be such that \( u_n \rightarrow u \) strongly in \( L^1(B)\) for all balls \(B\subset {\mathbb {R}}^N.\) Then,

$$\begin{aligned} \int _{{\mathbb {R}}^N}a |\textrm{D u}| \le \liminf _{n \rightarrow +\infty } \int _{{\mathbb {R}}^N}a |\mathrm{{Du}}_n|. \end{aligned}$$

Proof

Observe that if \( \displaystyle {\liminf _{n \rightarrow +\infty } \int _{{\mathbb {R}}^N}a |\mathrm{{Du}}_n| = + \infty ,} \) then the result is trivial. Hence, we can suppose that \( \displaystyle {\liminf _{n \rightarrow +\infty } \int _{{\mathbb {R}}^N}a |\mathrm{{Du}}_n| < + \infty .} \) Let B be a ball of \({\mathbb {R}}^N\) and \(\varphi \in C^{1}_{c}({\mathbb {R}}^N, {\mathbb {R}}^N)\) with compact support in B such that \(\left| \varphi (x)\right| \le 1,\ \forall x\in {\mathbb {R}}^N.\) We have

$$\begin{aligned} \int _{{\mathbb {R}}^N} u\ \text{ div }( a \varphi ) \textrm{d}x= & {} \lim _{n \rightarrow +\infty } \int _{{\mathbb {R}}^N} u_n\ \text{ div } (a \varphi ) \textrm{d}x = \liminf _{n \rightarrow +\infty } \int _{{\mathbb {R}}^N} u_n\ \text{ div } (a\varphi ) \textrm{d}x\\ {}\le & {} \liminf _{n \rightarrow +\infty } \int _{{\mathbb {R}}^N}a(x) |D u_n|. \end{aligned}$$

Now, we take the supremum over all such \( \varphi \), and the result immediately follows. \(\square \)

Proposition 1

Let B be an open ball of \( {\mathbb {R}}^N. \) The space \( \textrm{BV}^{b}_{a}( {\mathbb {R}}^N) \) is continuously embedded into the Lebesgue spaces \( L^t(B),\ \forall \ 1 \le t \le 1^*, \) where \(1^* =\frac{N}{N-1}. \)

Proof

Clearly, by \( (H_1), \) it yields

$$\begin{aligned} \textrm{BV}_a^b( {\mathbb {R}}^N) \subset \textrm{BV}(B) \end{aligned}$$

with continuous embedding. By [4, Theorem 10.1.3], we know that \( \textrm{BV}(B) \) is continuously embedded into \( L^t(B),\ \forall \ t \le 1 \le 1^*. \) This ends the proof of Proposition 1. \(\square \)

Remark 1

The embedding \(\textrm{BV}^{b}_{a}({\mathbb {R}}^N)\hookrightarrow L^t(B)\) is compact for \(1\le t < 1^*,\) for all balls \(B\subset {\mathbb {R}}^N.\) This result can be immediately deduced from [4, Theorem 10.1.4].

2.2 Weighted Anzellotti’s Theory

In the classical Anzellotti’s theory (see [3]), in order to define the pairing \( (z, \mathrm{{Du}}) \) as the distribution

$$\begin{aligned} \left\langle (z, \mathrm{{Du}}), \varphi \right\rangle = - \int _{\Omega } u^* \varphi \ \text{ div } z - \int _{\Omega } u z \cdot \nabla \varphi \textrm{d}x,\ \varphi \in D( \Omega ), \end{aligned}$$

where \( \Omega \) is an open set of \( {\mathbb {R}}^N,\ N \ge 2, \) and \( u^* \) is the precise representative of the function u (see [14]), we need some compatibility conditions such as \( u \in \textrm{BV}( \Omega ) \cap L^{\infty }( \Omega ) \) and \(\text{ div } z \in L^{1}( \Omega ) \) or \( u \in \textrm{BV}( \Omega ) \cap L^{\infty }( \Omega ) \cap C( \Omega ) \) and \( \text{ div } z \in {\mathcal {M}}( \Omega ). \) However, this definition has been extended in [7] to the case where \( u \in \textrm{BV}( \Omega ) \cap L^{\infty }( \Omega ) \) (i.e. the additional regularity condition \( u \in C( \Omega ) \) is removed) and \( \text{ div } z \in {\mathcal {M}}( \Omega ). \) Another further extension to the case where \( u \in \textrm{BV}_\textrm{loc}( \Omega ) \cap L^1( \Omega , \text{ div } z) \) and \(z \in \mathcal {D M}_{\textrm{loc}}^{\infty }( \Omega ) \) has been proved in [14]. We can also refer to [10,11,12].

In our present work, we need to establish a slightly further extension of the Anzellotti’s theory. For that aim, we define the space

$$\begin{aligned} {\mathcal {D}} {\mathcal {M}}^{\infty }_{a}({\mathbb {R}}^N)=\{ z\in L^{\infty }({\mathbb {R}}^N, {\mathbb {R}}^N) :\ \text{ div }(\textrm{az})\in {\mathcal {M}}({\mathbb {R}}^N)\}. \end{aligned}$$

and we reintroduce the space \({\mathcal {D}} {\mathcal {M}}^{\infty }_{a,\textrm{loc}}({\mathbb {R}}^N)\) defined as the space of all vector fields \(z\in L^{\infty }({\mathbb {R}}^N, {\mathbb {R}}^N)\) such that \(\text{ div }(\textrm{az})\in {\mathcal {M}}(B),\) for all balls \(B\subset {\mathbb {R}}^N.\)

Definition 2

Let \(z\in {\mathcal {D}}{\mathcal {M}}^{\infty }_{a, \textrm{loc}}({\mathbb {R}}^N)\) and \(u\in \textrm{BV}^{b}_{a}({\mathbb {R}}^N)\cap L^{\infty }({\mathbb {R}}^N).\) Then, we define the functional \(\Big ( \textrm{az},\mathrm{{Du}}\Big ):C^{\infty }_{c}({\mathbb {R}}^N)\rightarrow {\mathbb {R}}\) as

$$\begin{aligned} \begin{aligned} \Big <\Big ( \textrm{az},\mathrm{{Du}}\Big ),\varphi \Big >&=-\int _{{\mathbb {R}}^N} u^* \varphi \ \text{ div }(\textrm{a z}) -\int _{{\mathbb {R}}^N}\textrm{auz} \cdot \nabla \varphi (x) \textrm{d}x \\&= - \int _{{\mathbb {R}}^N} u^* \varphi d \nu - \int _{{\mathbb {R}}^N} \textrm{a u z} \cdot \nabla \varphi \textrm{d}x, \end{aligned} \end{aligned}$$

where \(\nu \) is the regular Borel measure that represents \( \text{ div }(a z). \)

Proposition 2

Let \(z\in {\mathcal {D}} {\mathcal {M}}^{\infty }_{a, \textrm{loc}}({\mathbb {R}}^N)\) and \(u\in \textrm{BV}_{a}^{b}({\mathbb {R}}^N)\cap L^{\infty }({\mathbb {R}}^N). \) Then,

$$\begin{aligned} \Big | \Big <\left( \textrm{az},\mathrm{{Du}}\right) ,\ \varphi \Big >\Big |\le \left| \varphi \right| _{\infty }\left| z\right| _{L^{\infty }({\mathbb {R}}^N)}\int _{ {\mathbb {R}}^N} a|\mathrm{{Du}}|. \end{aligned}$$
(2.1)

Proof

Fix \(u \in \textrm{BV}_{a}^{b}({\mathbb {R}}^N)\cap L^{\infty }({\mathbb {R}}^N).\) Let \( (\rho _n)_n \) be a sequence of mollifiers and define the sequence \( (u_n)_n \subset C^{\infty }( {\mathbb {R}}^N) \cap L^{\infty }( {\mathbb {R}}^N) \) by \( u_n = \rho _n * u,\ n \ge 1. \) Plainly, \( u_n(x) \rightarrow u(x) \) a.e. \( x \in {\mathbb {R}}^N \) and \( u_n(x) \rightarrow u^*(x),\ {\mathcal {H}}^{N-1}-\)a.e. \( x \in {\mathbb {R}}^N, \) where \( {\mathcal {H}}^{N-1} \) stands for the \( (N-1)-\)dimensional Hausdorff measure in \( {\mathbb {R}}^N. \) Moreover, \( (u_n)_n \) is bounded in \( L^{\infty }( {\mathbb {R}}^N). \) It is easy to show that

$$\begin{aligned} \int _{{\mathbb {R}}^N} b \left| u_n\right| \textrm{d}x \rightarrow \int _{{\mathbb {R}}^N} b \left| u\right| \textrm{d}x, \end{aligned}$$

and

$$\begin{aligned} \int _{B} a \left| \nabla u_n\right| \textrm{d}x \rightarrow \int _B a \left| \mathrm{{Du}}\right| ,\ \text{ for } \text{ all } \text{ ball }\ B\ \text{ of }\ {\mathbb {R}}^N, \end{aligned}$$

as \( n \rightarrow + \infty . \) Taking into account that \( \left| \text{ div }(\textrm{az})\right|<< {\mathcal {H}}^{N-1} \) (see [7]), then one can apply the Lebesgue’s dominated convergence theorem with respect to the Radon measure \( \text{ div }(\textrm{az}), \) to obtain that

$$\begin{aligned} \int _{{\mathbb {R}}^N} u_n \varphi \ \text{ div } (\textrm{a z}) \rightarrow \int _{{\mathbb {R}}^N} u^* \varphi \ \text{ div }(\textrm{a z}). \end{aligned}$$

In a similar way, applying the Lebesgue’s dominated convergence Theorem with respect to the Lebesgue measure, we also get

$$\begin{aligned} \int _{{\mathbb {R}}^N} u_n \textrm{a z} \cdot \nabla \varphi \textrm{d}x \rightarrow \int _{{\mathbb {R}}^N} u \textrm{a z}\cdot \nabla \varphi \textrm{d}x. \end{aligned}$$

Hence,

$$\begin{aligned} \left\langle (\textrm{a z}, \mathrm{{Du}}_n), \varphi \right\rangle \rightarrow -\int _{{\mathbb {R}}^N} u^* \varphi \ \text{ div }(\textrm{a z}) -\int _{{\mathbb {R}}^N} \textrm{u a z}\cdot \nabla \varphi \textrm{d}x = \left\langle (\textrm{a z}, \mathrm{{Du}}), \varphi \right\rangle . \end{aligned}$$
(2.2)

On the other hand, let B be a ball such that \( \text{ supp }( \varphi ) \subset B. \) Having in mind that,

$$\begin{aligned} \int _{{\mathbb {R}}^N} u_n \varphi \ \text{ div }(\textrm{a z}) = - \int _{{\mathbb {R}}^N} a \varphi z \cdot \nabla u_n \textrm{d}x - \int _{{\mathbb {R}}^N} a u_n z \cdot \nabla \varphi \textrm{d}x, \end{aligned}$$

we infer

$$\begin{aligned} \left| \left\langle (a z, \mathrm{{Du}}_n), \varphi \right\rangle \right| = \left| \int _{{\mathbb {R}}^N} \varphi a z \cdot \nabla u_n \textrm{d}x\right| = \left| \int _B \varphi \textrm{a z} \cdot \nabla u_n \textrm{d}x \right| \le \left| \varphi \right| _{\infty } \left| z\right| _{L^{\infty }( {\mathbb {R}}^N)} \int _B a \left| \nabla u_n \right| \textrm{d}x. \end{aligned}$$

Passing to the limit as \( n \rightarrow + \infty , \) in that last inequality, and taking (2.2) into account, we deduce that (2.1) holds true. \(\square \)

Corollary 1

The distribution \((a z,\mathrm{{Du}})\) is a Radon measure and its total variation \(|(a z,\mathrm{{Du}})|\) are absolutely continuous with respect to the measure \(a|\mathrm{{Du}}|\) and

$$\begin{aligned} \Big |\int _{B}\Big ( \textrm{az},\mathrm{{Du}}\Big )\Big |\le \int _{B}\Big |\Big ( \textrm{az},\mathrm{{Du}}\Big )\Big |\le \left| z\right| _{L^{\infty }({\mathbb {R}}^N)}\int _{B} a|\mathrm{{Du}}| \end{aligned}$$
(2.3)

holds for all Borel sets \(B \subseteq {\mathbb {R}}^N.\)

Next, according to Definition 2, we can easily see that the following proposition holds.

Proposition 3

Let \(z\in {\mathcal {D}} {\mathcal {M}}^{\infty }_{a,\textrm{loc}}({\mathbb {R}}^N)\) and \( u \in \textrm{BV}^{b}_{a}({\mathbb {R}}^N)\cap L^{\infty }({\mathbb {R}}^N).\) Then, \(\textrm{az} u\in {\mathcal {D}} {\mathcal {M}}^{\infty }_{a, \textrm{loc}}({\mathbb {R}}^N).\) Moreover the following formula holds in the sense of measures

$$\begin{aligned} \text{ div }(u \textrm{az})=(\textrm{az}, \mathrm{{Du}})+u^*\text{ div }(\textrm{az}). \end{aligned}$$

Finally, we prove that the following Gauss–Green’s formula holds.

Theorem 2.2

Let \( u \in \textrm{BV}_a^b( {\mathbb {R}}^N) \cap L^{\infty }( {\mathbb {R}}^N) \) and \( z \in L^{\infty }( {\mathbb {R}}^N,{\mathbb {R}}^N) \) be such that \( \text{ div }(\textrm{az}) \in L^1( {\mathbb {R}}^N). \) Then, we have

$$\begin{aligned} \int _{{\mathbb {R}}^N} (\textrm{az}, \mathrm{{Du}}) = - \int _{{\mathbb {R}}^N} u \text{ div }(a z) \textrm{d}x. \end{aligned}$$

Proof

Consider a cutoff function \( \theta \in C_c^{\infty }( {\mathbb {R}}^N)\) such that \( 0 \le \theta \le 1,\ \theta (x) = 1,\ \forall \ \left| x\right| \le 1, \) and \( \theta (x) = 0,\ \forall \ \left| x\right| \ge 2. \) For \( k=1,2,3,\cdots , \) define the function \( \theta _k: {\mathbb {R}}^N \rightarrow {\mathbb {R}} \) by

$$\begin{aligned} \theta _k(x) = \theta \left( \frac{x}{k}\right) ,\ x \in {\mathbb {R}}^N. \end{aligned}$$

Since \( \text{ supp }( \theta _k) \) is compact, then \( \theta _k u \in \textrm{BV}( {\mathbb {R}}^N). \) Taking into account that \( \text{ div }(\textrm{az}) \in L^1( {\mathbb {R}}^N), \) one can apply the classical Gauss–Green’s formula ( [3]), that is

$$\begin{aligned} \int _{{\mathbb {R}}^N} (\textrm{az}, D( \theta _ku)) = - \int _{{\mathbb {R}}^N} \theta _k u \text{ div }(\textrm{az}) \textrm{d}x. \end{aligned}$$

Having in mind that

$$\begin{aligned} (\textrm{az}, D(\theta _ku)) = \theta _k(\textrm{az},\mathrm{{Du}})+\textrm{uaz} \cdot \nabla \theta _k,\ \text{ as } \text{ Radon } \text{ measures }, \end{aligned}$$

it yields

$$\begin{aligned} \int _{{\mathbb {R}}^N} \theta _k(\textrm{az, Du})+\int _{{\mathbb {R}}^N}u \textrm{az} \cdot \nabla \theta _k \textrm{d}x = - \int _{{\mathbb {R}}^N} \theta _k u \text{ div }(az) \textrm{d}x. \end{aligned}$$
(2.4)

Observing that \( \theta _k(x) \rightarrow 1,\ \forall \ x \in {\mathbb {R}}^N \) and \( 0 \le \theta _k(x) \le 1,\ \forall \ x \in {\mathbb {R}}^N,\ \forall \ k=1,2,3,\cdots , \) , then by the Lebesgue-dominated convergence theorem, it follows that

$$\begin{aligned} \lim _{k \rightarrow \infty }\int _{{\mathbb {R}}^N} \theta _k(\textrm{az}, \mathrm{{Du}}) = \int _{{\mathbb {R}}^N} (az, \mathrm{{Du}}) \end{aligned}$$
(2.5)

and

$$\begin{aligned} \lim _{k \rightarrow \infty } \int _{{\mathbb {R}}^N} \theta _k u \text{ div }(\textrm{az}) \textrm{d}x= \int _{{\mathbb {R}}^N} u \text{ div }(\textrm{az}) \textrm{d}x. \end{aligned}$$
(2.6)

On the other hand, we have

$$\begin{aligned} \left| \int _{{\mathbb {R}}^N}u \textrm{az} \cdot \nabla \theta _k \textrm{d}x\right| \le \frac{1}{k} |z|_{L^{\infty }( {\mathbb {R}}^N)}|u|_{L^{\infty }( {\mathbb {R}}^N)} \left| \nabla \theta \right| _{L^{\infty }( {\mathbb {R}}^N)} \left| a\right| _{L^1( {\mathbb {R}}^N)} \rightarrow 0,\ k \rightarrow +\infty .\nonumber \\ \end{aligned}$$
(2.7)

Combining (2.7), (2.6) and (2.5) with (2.4), we obtain

$$\begin{aligned} \int _{{\mathbb {R}}^N} (\textrm{az}, \mathrm{{Du}}) = - \int _{{\mathbb {R}}^N} u \text{ div }(\textrm{az}) \textrm{d}x. \end{aligned}$$

\(\square \)

2.3 The Space \(W^{1,p}_{a,b}({\mathbb {R}}^{N})\)

Taking into account the fact that our existence result will be established through some approximation scheme, some properties of the weighted Sobolev space \( W^{1,p}_{a,b}( {\mathbb {R}}^N) \) have to be stated. This is the object of the present subsection.

Let \(1<p<2.\) Set,

$$\begin{aligned} W^{1,p}_{a,b}({\mathbb {R}}^{N})=\left\{ u:{\mathbb {R}}^N \rightarrow {\mathbb {R}}\ \text{ measurable };\ \int _{{\mathbb {R}}^{N}}b|u|^p\textrm{d}x<+ \infty ,\ \int _{{\mathbb {R}}^{N}}a|\nabla u|^p\textrm{d}x< + \infty \right\} . \end{aligned}$$

This space is provided with the norm

$$\begin{aligned} \Vert u\Vert _{W^{1,p}_{a,b}({\mathbb {R}}^{N})}=\Big [\int _{{\mathbb {R}}^{N}}b|u|^p\textrm{d}x+\int _{{\mathbb {R}}^{N}}a|\nabla u|^p\textrm{d}x\Big ]^\frac{1}{p}. \end{aligned}$$

Since \(a,\ b \in L^1_{\textrm{loc}}({\mathbb {R}}^{N}),\) and \(a^{-1/(p-1)} ,\ b^{-1/(p-1)}\in L^1_{\textrm{loc}}({\mathbb {R}}^{N}),\) then \(\Big (W^{1,p}_{a,b}({\mathbb {R}}^{N}), \Vert .\Vert _{W^{1,p}_{a,b}({\mathbb {R}}^{N})}\Big )\) becomes a Banach, reflexive and separable space (see [28, 29]).

Lemma 2

Let \( 1< p < 2. \) The embedding \(W^{1,p}_{a,b}({\mathbb {R}}^N)\hookrightarrow W^{1,1}_{a,b}({\mathbb {R}}^N)\) is continuous. Precisely, there exists a constant \(C> 0\) independent of p such that

$$\begin{aligned} \Vert u\Vert _{W^{1,1}_{a,b}({\mathbb {R}}^N)}\le C \Vert u\Vert _{W^{1,p}_{a,b}({\mathbb {R}}^N)},\ \forall u\in W^{1,p}_{a,b}({\mathbb {R}}^N), \end{aligned}$$
(2.8)

where

$$\begin{aligned} W^{1,1}_{a,b}({\mathbb {R}}^{N})\!=\!\{u:{\mathbb {R}}^N \!\rightarrow \! {\mathbb {R}}\ \text{ measurable }; \int _{{\mathbb {R}}^{N}}b |u|\textrm{d}x<+ \infty , \int _{{\mathbb {R}}^{N}} a|\nabla u|\textrm{d}x< + \infty \}, \end{aligned}$$

and

$$\begin{aligned} \Vert u\Vert _{W^{1,1}_{a,b}({\mathbb {R}}^N)} = \int _{{\mathbb {R}}^N} a \left| \nabla u\right| \textrm{d}x + \int _{{\mathbb {R}}^N} b \left| u\right| \textrm{d}x. \end{aligned}$$

Proof

Let \(u\in W^{1,p}_{a,b}({\mathbb {R}}^N).\) By \((H_1)\) and the Hölder’s inequality, we have

$$\begin{aligned} \Vert u\Vert _{W^{1,1}_{a,b}({\mathbb {R}}^N)}= & {} \int _{{\mathbb {R}}^N}a|\nabla u|\textrm{d}x+ \int _{{\mathbb {R}}^N}b|u|\textrm{d}x\\= & {} \int _{{\mathbb {R}}^N}a^{\frac{p-1}{p}}a^{\frac{1}{p}}|\nabla u|\textrm{d}x+ \int _{{\mathbb {R}}^N}b^{\frac{p-1}{p}}b^{\frac{1}{p}}|u|\textrm{d}x\\\le & {} \Big (\int _{{\mathbb {R}}^N}a\textrm{d}x\Big )^{\frac{p-1}{p}} \Big ( \int _{{\mathbb {R}}^N}a|\nabla u|^p\textrm{d}x\Big )^{\frac{1}{p}} + \Big (\int _{{\mathbb {R}}^N}b\textrm{d}x\Big )^{\frac{p-1}{p}} \Big ( \int _{{\mathbb {R}}^N}b| u|^p\textrm{d}x\Big )^{\frac{1}{p}}\\\le & {} \left( |b|^{\frac{p-1}{p}}_{L^1({\mathbb {R}}^N)}+|a|^{\frac{p-1}{p}}_{L^1({\mathbb {R}}^N)}\right) \Vert u\Vert _{W^{1,p}_{a,b}({\mathbb {R}}^N)}.\\\le & {} \Big (\left( |b|_{L^1({\mathbb {R}}^N)}+1\right) ^{\frac{p-1}{p}}+\left( |a|_{L^1({\mathbb {R}}^N)}+1\right) ^{\frac{p-1}{p}}\Big ) \Vert u\Vert _{W^{1,p}_{a,b}({\mathbb {R}}^N)}\\\le & {} \left( 2+|b|_{L^1({\mathbb {R}}^N)}+|a|_{L^1({\mathbb {R}}^N)}\right) \Vert u\Vert _{W^{1,p}_{a,b}({\mathbb {R}}^N)}\\\le & {} C \Vert u\Vert _{W^{1,p}_{a,b}({\mathbb {R}}^N)}. \end{aligned}$$

\(\square \)

Lemma 3

For all \(R>0,\) there exists a constant \(C_R>0\) independent of p such that

$$\begin{aligned} \Vert u\Vert _{W^{1,1}(B(0,R))}\le C_R \Vert u\Vert _{W_{a,b}^{1,p}({\mathbb {R}}^N)},\ \forall u\in W^{1,p}_{a,b}({\mathbb {R}}^N), \end{aligned}$$
(2.9)

where \(B(0,R)=\{x\in {\mathbb {R}}^N,\ |x|<R\}.\)

Proof

Let \( R > 0 \) and \(u\in W_{a,b}^{1,p}({\mathbb {R}}^N).\) By \((H_{1}), \) we have

$$\begin{aligned} \Vert u\Vert _{W^{1,1}(B(0,R))}= & {} \int _{B(0,R)}|u|\textrm{d}x+\int _{B(0,R)}|\nabla u|\textrm{d}x\\\le & {} \frac{1}{b_R}\int _{B(0,R)}b|u|\textrm{d}x+\frac{1}{a_R}\int _{B(0,R)}a|\nabla u|\textrm{d}x\\\le & {} \frac{1}{b_R}\int _{{\mathbb {R}}^N}b|u|\textrm{d}x+\frac{1}{b_R}\int _{{\mathbb {R}}^N}a|\nabla u|\textrm{d}x\\\le & {} \max \left( \frac{1}{b_R}+\frac{1}{a_R} \right) \Vert u\Vert _{W^{1,1}_{a,b}({\mathbb {R}}^N)}. \end{aligned}$$

Therefore, inequality (2.9) can be deduced from Lemma 1. \(\square \)

Remark 2

One of the consequences of Lemma 3 is that the following continuous embedding holds

$$\begin{aligned} W^{1,p}_{a,b}({\mathbb {R}}^N)\hookrightarrow L^t(B(0,R)),\ \forall \ R > 0,\ \forall \ t \in [1,1^*],\ \text{ where }\ 1^* = \frac{N}{N-1}. \end{aligned}$$

Moreover, by the virtue of the Rellich–Kondrachov compactness Theorem, we can immediately deduce that the embedding \(W^{1,p}_{a,b}({\mathbb {R}}^{N})\hookrightarrow L^t(B(0,R))\) is compact for all \(1\le t < 1^*,\ \forall \ R>0.\)

Now, we prove that \( C^{\infty }_c( {\mathbb {R}}^N)\) is dense in \(W^{1,1}_{a,b}( {\mathbb {R}}^N).\)

Proposition 4

The space \( C^{\infty }_c( {\mathbb {R}}^N) \) is dense in \( W^{1,1}_{a,b}( {\mathbb {R}}^N) \).

Proof

Let \( v\in W^{1,1}_{a,b}( {\mathbb {R}}^N) \) and \( \phi \in C_c^{\infty }( {\mathbb {R}}^N) \) be a cutoff function such that \( \phi (x) = 1,\ \forall \ \left| x\right| \le 1,\ \phi (x) = 0,\ \forall \ \left| x\right| \ge 2, \) and \( 0 \le \phi \le 1. \) For \( k \in {\mathbb {N}}^* \) and \( x \in {\mathbb {R}}^N, \) set \( \phi _k(x) = \phi \left( \frac{x}{k}\right) \) and \( v_k(x) = \phi _k(x) v(x). \) We claim that \(v_k \rightarrow v,\ k \rightarrow + \infty , \) strongly in \(W^{1,1}_{a,b}( {\mathbb {R}}^N). \) We have

$$\begin{aligned} \int _{{\mathbb {R}}^N} b|v_k - v| \textrm{d}x= \int _{{\mathbb {R}}^N} b|\phi _k(x) -1||v| \textrm{d}x. \end{aligned}$$

Observing that \(|\phi _k(x) -1 | \rightarrow 0,\ \forall x\in {\mathbb {R}}^N, \) and that \(|\phi _k(x) -1|\le 2,\ \forall x\in {\mathbb {R}}^N, \) then by the Lebesgue dominated convergence Theorem, it follows that

$$\begin{aligned} \lim _{k \rightarrow +\infty } \int _{{\mathbb {R}}^N} b|v_k - v| \textrm{d}x=0. \end{aligned}$$
(2.10)

On the other hand,

$$\begin{aligned} \begin{aligned}&\int _{{\mathbb {R}}^N} a|\nabla v_k - \nabla v|\textrm{d}x\\&\quad =\int _{{\mathbb {R}}^N} a|\left( \phi _k -1\right) \nabla v + v\nabla \phi _k| \textrm{d}x\\&\quad \le \int _{{\mathbb {R}}^N} a\left| \phi _k -1 \right| \left| \nabla v\right| \textrm{d}x + \int _{{\mathbb {R}}^N} a\left| v\right| \left( \frac{1}{k} \right) \left| \nabla \phi \left( \frac{x}{k} \right) \right| \textrm{d}x\\&\quad \le \int _{{\mathbb {R}}^N} a\left| \phi _k - 1\right| \left| \nabla v\right| \textrm{d}x + C_0 \int _{{\mathbb {R}}^N} b\left| v\right| \left( \frac{1}{k} \right) \left| \nabla \phi \left( \frac{x}{k} \right) \right| \textrm{d}x, \end{aligned} \end{aligned}$$
(2.11)

where we used the fact that \( a(x)\le C_0 b(x),\ \forall x\in {\mathbb {R}}^N. \) Plainly,

$$\begin{aligned} \int _{{\mathbb {R}}^N} b\left| v\right| \left( \frac{1}{k} \right) \left| \nabla \phi \left( \frac{x}{k} \right) \right| \textrm{d}x \rightarrow 0,\ k \rightarrow +\infty . \end{aligned}$$
(2.12)

Again by the virtue of the Lebesgue-dominated convergence theorem, we get

$$\begin{aligned} \lim _{k \rightarrow +\infty } \int _{{\mathbb {R}}^N} a\left| \phi _k - 1 \right| \left| \nabla v\right| \textrm{d}x=0. \end{aligned}$$
(2.13)

By (2.13) and (2.12), we infer

$$\begin{aligned} \lim _{k \rightarrow +\infty } \int _{{\mathbb {R}}^N} a|\nabla v_k - \nabla v|\textrm{d}x=0. \end{aligned}$$
(2.14)

Combining (2.14) and (2.10), we deduce that \(v_k \rightarrow v\) strongly in \(W^{1,1}_{a,b}( {\mathbb {R}}^N). \) Hence, for every \( \epsilon > 0, \) there exists \( n_0 = n_0(\epsilon ) \ge 1 \) such that

$$\begin{aligned} \left\| v_{n_0} - v\right\| _{W^{1,1}_{a,b}( {\mathbb {R}}^N)} \le \frac{\epsilon }{2}. \end{aligned}$$
(2.15)

Now, recall that

$$\begin{aligned} 0< \inf _{\left| x\right|< 2n_0} a(x)< + \infty ,\ \text{ and }\ 0< \inf _{\left| x\right|< 2n_0} b(x) < + \infty . \end{aligned}$$

Then,

$$\begin{aligned}{} & {} \left\{ u \in L^1(B(0,2n_0)),\ \int _{B(0,2n_0)}a \left| \nabla u\right| \textrm{d}x< + \infty ,\ \int _{B(0,2n_0)} b \left| u\right| \textrm{d}x < + \infty \right\} \\ {}{} & {} \quad = W^{1,1}(B(0,2n_0)). \end{aligned}$$

Having in mind that \( v_{n_0} \in W_0^{1,1}(B(0,2n_0)) \) and that \( W_0^{1,1}(B(0,2n_0)) \hookrightarrow W^{1,1}_{a,b}( {\mathbb {R}}^N), \) we infer that there exists \( {\widetilde{v}} \in C_c^{\infty }(B(0,2n_0)) \subset C_c^{\infty }( {\mathbb {R}}^N) \) such that

$$\begin{aligned} \left\| v_{n_0} - {\widetilde{v}}\right\| _{W^{1,1}_{a,b}( {\mathbb {R}}^N)} \le \frac{\epsilon }{2}. \end{aligned}$$
(2.16)

Combining (2.15) and (2.16), we obtain the claimed result. \(\square \)

3 Weak Solution to the Approximated p-Laplacian Problem

For \(1< p < 2,\) let us consider the following problem

$$\begin{aligned} -\textrm{div} \Big (a|\nabla u|^{p-2}\nabla u\Big )+b|u|^{p-2}u= \frac{h}{\left( u^+\right) ^{1-q}}&\text{ in }~~{\mathbb {R}}^{N} ,\ N\ge 2, \end{aligned}$$
(3.1)

where \( u^+ = \max (u,0). \) We define the functional

$$\begin{aligned} J_p(u)=\frac{1}{p}\Vert u\Vert ^p_{W^{1,p}_{a,b}({\mathbb {R}}^N)}-\frac{1}{q}\int _{{\mathbb {R}}^N}h (u^+)^q \textrm{d}x. \end{aligned}$$

It is well known that the singular term leads to the non-differentiability of the functional \(J_p\) on \(W^{1,p}_{a,b}({\mathbb {R}}^N), \) so \(J_p\) does not belong to \(C^1(W^{1,p}_{a,b}({\mathbb {R}}^N),\ {\mathbb {R}}). \) Therefore, the problem cannot be considered by using directly the classical critical point theory. Nevertheless, we shall prove that problem (3.1) still has a solution, which is a local minimizer of the energy functional \(J_p.\) For \( n=1,2,3,\cdots , \) let us consider the following approximated problem:

$$\begin{aligned} -\textrm{div} \Big (a|\nabla u|^{p-2}\nabla u\Big )+b|u|^{p-2}u= \frac{h}{(u^+ + \frac{1}{n})^{1-q}}&\text{ in }~~{\mathbb {R}}^{N} ,\ N\ge 2, \end{aligned}$$
(3.2)

and its corresponding energy functional:

$$\begin{aligned} J_{n,p}(u)=\frac{1}{p}\Vert u\Vert _{W^{1,p}_{a,b}({\mathbb {R}}^N)}^p -\int _{{\mathbb {R}}^N}h \chi _n(u) \textrm{d}x,\ u\in W^{1,p}_{a,b}({\mathbb {R}}^N), \end{aligned}$$

where

$$\begin{aligned} \chi _n(s)=\int _{0}^{s}\left( t^+ + \frac{1}{n} \right) ^{q-1}dt = \frac{\left( s^+ +\frac{1}{n} \right) ^q -\left( \frac{1}{n} \right) ^q}{q} + \left( \frac{1}{n}\right) ^{q-1}s^-,\ s\in {\mathbb {R}}, \end{aligned}$$

with \(s^-=\min \{s,0\}. \) By studying deeply the functional \(J_{n,p},\) we can deduce the existence of solution to our original problem (3.1) for \( 1< p < 2.\)

Definition 3

A function \(u_p\in W^{1,p}_{a,b}({\mathbb {R}}^N) \) is said to be a weak solution of (3.1) if it satisfies that \(u_p(x)> 0 \) a.e. \( x \in {\mathbb {R}}^N,\ \frac{h v}{u_p^{1-q}} \in L^1({\mathbb {R}}^N),\ \forall v\in W^{1,p}_{a,b}({\mathbb {R}}^N),\) and

$$\begin{aligned} \int _{{\mathbb {R}}^N}a|\nabla u_p|^{p-2}\nabla u_p.\nabla v \textrm{d}x+\int _{{\mathbb {R}}^N}b|u_p|^{p-2} u_p v \textrm{d}x=\int _{{\mathbb {R}}^N}\frac{h\ v}{u_p^{1-q}}\textrm{d}x,\ \forall v\in W^{1,p}_{a,b}({\mathbb {R}}^N). \end{aligned}$$

Theorem 3.1

Assume that \((H_1)\) and \((H_2)\) hold. Then, for any fixed \( 1< p < 2, \) there exists a bounded weak solution to (3.1) in the sense of Definition (3).

The proof of Theorem (3.1) will be divided in several steps.

Lemma 4

For \(1<p<2,\) there exist r and \( \rho >0 \) independent of p such that

$$\begin{aligned} J_p(u) \ge \rho ,\ \forall u\in W^{1,p}_{a,b}({\mathbb {R}}^N),\ \Vert u\Vert _{W^{1,p}_{a,b}({\mathbb {R}}^N)}=r. \end{aligned}$$

Moreover,

$$\begin{aligned} m_p=\inf _{ \Vert u\Vert _{W^{1,p}_{a,b}({\mathbb {R}}^N)}\le r} J_p(u)<0. \end{aligned}$$

Proof

By Lemma 2, Hölder’s inequality and \((H_2),\) for \( u \in W^{1,p}_{a,b}( {\mathbb {R}}^N),\) we have

$$\begin{aligned} \begin{aligned} \frac{1}{q}\int _{{\mathbb {R}}^N}h (u^+)^q \textrm{d}x= \frac{1}{q}\int _{{\mathbb {R}}^N}\frac{h}{b^q} b^q (u^+)^q \textrm{d}x&\le \frac{1}{q}|\frac{h}{b^q}|_{L^{\frac{1}{1-q}}( {\mathbb {R}}^N)}\Vert u\Vert _{W^{1,1}_{a,b}( {\mathbb {R}}^N)}^q\\&\le C \frac{1}{q}|\frac{h}{b^q}|_{L^{\frac{1}{1-q}}( {\mathbb {R}}^N)} \Vert u\Vert ^q_{W^{1,p}_{a,b}( {\mathbb {R}}^N)} \\ {}&\le C_1 \Vert u\Vert ^q_{W^{1,p}( {\mathbb {R}}^N)}. \end{aligned} \end{aligned}$$

Recall that the constant C given in Lemma 2 is independent of p. It yields,

$$\begin{aligned} \begin{aligned} J_p(u)&=\frac{1}{p}\Vert u\Vert _{W^{1,p}_{a,b}({\mathbb {R}}^N)}^p-\frac{1}{q}\int _{{\mathbb {R}}^N}h (u^+)^q \textrm{d}x\\&\ge \frac{1}{p}\Vert u\Vert ^p_{W^{1,p}_{a,b}({\mathbb {R}}^N)}- C_1 \Vert u\Vert ^q_{W^{1,p}_{a,b}( {\mathbb {R}}^N)}\\&\ge \Vert u\Vert _{W^{1,p}_{a,b}({\mathbb {R}}^N)}^q\left( \frac{1}{p}\Vert u\Vert _{W^{1,p}_{a,b}({\mathbb {R}}^N)}^{p-q}- C_1\right) . \end{aligned} \end{aligned}$$
(3.3)

Choose \(r>0\) small enough such that \(\frac{r^{p-q}}{p} -C_1\ge \frac{r^{2-q}}{2} -C_1 > 0,\ \forall \ 1<p<2.\) By (3.3), we infer

$$\begin{aligned} J_p(u)\ge & {} \rho =r^q\left( r^{2-q}- C_1\right) ,\ u\in W^{1,p}_{a,b}({\mathbb {R}}^N),\ \Vert u\Vert _{W^{1,p}_{a,b}({\mathbb {R}}^N)}=r. \end{aligned}$$

Now, let \(\varphi \in W^{1,p}_{a,b}({\mathbb {R}}^N)\) be such that \(\varphi > 0\) and \(\Vert \varphi \Vert _{W^{1,p}_{a,b}({\mathbb {R}}^N)}\le r.\) For \(t>0, \) we get

$$\begin{aligned} \lim _{t \rightarrow 0^+}\frac{J_p(t\varphi )}{t^q}= -\frac{1}{q} \int _{{\mathbb {R}}^N}h \varphi ^q \textrm{d}x<0. \end{aligned}$$

By consequence, one can easily find \(0<t_0<1\) small enough such that \(\Vert t_0 \varphi \Vert _{W^{1,p}_{a,b}({\mathbb {R}}^N)}\le r\) and \(J_p(t_0\varphi )<0. \) We conclude that \(m_p\le J_p(t_0\varphi )<0.\) This ends the proof of Lemma (4). \(\square \)

Lemma 5

The problem (3.1) has a positive solution \(u_p \in W^{1,p}_{a,b}({\mathbb {R}}^N) \) satisfying that \(J_p(u_p)<0. \)

Proof

First, we claim that there exists \(u_p \in W^{1,p}_{a,b}({\mathbb {R}}^N)\) such that \(\Vert u_p\Vert _{W^{1,p}_{a,b}({\mathbb {R}}^N)}\le r\) and \(J_p(u_p)=m_p<0.\)

For that aim, fix \(n\ge 1. \) Observe that since \(0<q<1, \) then

$$\begin{aligned} J_{n,p}(u)\ge J_p(u),\ \forall \ u\in W^{1,p}_{a,b}({\mathbb {R}}^N),\ \forall \ n\ge 1. \end{aligned}$$

By Lemma 4, it yields

$$\begin{aligned} J_{n,p}(u) \ge \rho >0,\ \forall u\in W^{1,p}_{a,b}({\mathbb {R}}^N),\ \Vert u\Vert _{W^{1,p}_{a,b}({\mathbb {R}}^N)}=r. \end{aligned}$$

Consequently,

$$\begin{aligned} \inf _{\Vert u\Vert _{W^{1,p}_{a,b}({\mathbb {R}}^N)}=r} J_{n,p}(u)>\inf _{\Vert u\Vert _{W^{1,p}_{a,b}({\mathbb {R}}^N)}<r} J_{n,p}(u)\le J_{n,p}(0)=0. \end{aligned}$$

Since \(J_{n,p}\) is of class \(C^1\) on \( W^{1,p}_{a,b}({\mathbb {R}}^N),\) then we can apply the Ekeland variational principle (see [16]) to construct a sequence \((u_{j,n,p})_j\subset W^{1,p}_{a,b}({\mathbb {R}}^N)\) such that \(\Vert u_{j,n,p}\Vert _{W^{1,p}_{a,b}( {\mathbb {R}}^N)}\le r,\ J_{n,p}(u_{j,n,p}) \rightarrow m_{n,p}= \inf \limits _{\Vert u\Vert _{W^{1,p}_{a,b}( {\mathbb {R}}^N)}\le r} J_{n,p}(u), \) and \( J'_{n,p}(u_{j,n,p}) \rightarrow 0,\) as \(j \rightarrow + \infty .\) Since \((u_{j,n,p})_j\) is bounded in \(W^{1,p}_{a,b}({\mathbb {R}}^N),\) then there exists \(u_{n,p}\in W^{1,p}_{a,b}({\mathbb {R}}^N)\) such that \(u_{j,n,p}\rightharpoonup u_{n,p}\) weakly in \(W^{1,p}_{a,b}({\mathbb {R}}^N)\) and \(u_{j,n,p}(x)\rightarrow u_{n,p}(x) \) a.e. \( x\in {\mathbb {R}}^N.\) This implies that \(b^{\frac{1}{p}}u_{j,n,p}\rightharpoonup b^{\frac{1}{p}} u_{n,p}\) weakly in \(L^p({\mathbb {R}}^N)\) and \(b^{\frac{1}{p}} (x)u_{j,n,p}(x)\rightarrow b^{\frac{1}{p}}(x)u_{n,p}(x), \) a.e. \( x\in {\mathbb {R}}^N.\) Set \(w_{j,n,p}=u_{j,n,p} -u_{n,p}. \) By Brezis–Lieb’s lemma ( see [26, Chapter 1, Lemma 4.6]), one has

$$\begin{aligned} \int _{{\mathbb {R}}^N}b| u_{j,n,p} |^p \textrm{d}x =\int _{{\mathbb {R}}^N}b|w_{j,n,p} |^p \textrm{d}x+\int _{{\mathbb {R}}^N}b| u_{n,p} |^p \textrm{d}x+o_j(1), \end{aligned}$$
(3.4)

where \(o_j(1)\) denotes a sequence of real numbers tending to 0 when j tends to \(+\infty .\)

Now, we claim that, for a.e. \( x \in {\mathbb {R}}^N,\ a(x)^{\frac{1}{p}}\nabla u_{j,n,p}(x) \rightarrow a(x)^{\frac{1}{p}}\nabla u_{n,p}(x),\) as \(j \rightarrow \infty .\) Since \(\left\langle J'_{n,p}(u_{j,n,p}), w_{j,n,p} \right\rangle \rightarrow 0,\) as \(j \rightarrow +\infty ,\) then

$$\begin{aligned} \begin{aligned}&\int _{{\mathbb {R}}^N}a|\nabla u_{j,n,p}|^{p-2}\nabla u_{j,n,p} \nabla w_{j,n,p} \textrm{d}x+ \int _{{\mathbb {R}}^N}b|u_{j,n,p}|^{p-2} u_{j,n,p} w_{j,n,p}\textrm{d}x\\&\quad =o_j(1)+\int _{{\mathbb {R}}^N}h\left( u^+_{j,n,p}+\frac{1}{n} \right) ^{q-1} w_{j,n,p} \textrm{d}x. \end{aligned} \end{aligned}$$
(3.5)

We have

$$\begin{aligned} \begin{aligned}&\int _{{\mathbb {R}}^N}a|\nabla u_{j,n,p}|^{p-2}\nabla u_{j,n,p} \nabla w_{j,n,p}\textrm{d}x\\ {}&\quad =\int _{{\mathbb {R}}^N}a\left( |\nabla u_{j,n,p}|^{p-2}\nabla u_{j,n,p}-|\nabla u_{n,p}|^{p-2}\nabla u_{n,p}\right) \nabla w_{j,n,p}\textrm{d}x\\&\qquad +\int _{{\mathbb {R}}^N}a|\nabla u_{n,p}|^{p-2}\nabla u_{n,p} \nabla w_{j,n,p}\textrm{d}x. \end{aligned} \end{aligned}$$
(3.6)

By the weak convergence of \((w_{j,n,p})_j\) to 0 in \(W^{1,p}_{a,b}({\mathbb {R}}^N),\) we infer

$$\begin{aligned} \int _{{\mathbb {R}}^N}a|\nabla u_{n,p}|^{p-2}\nabla u_{n,p} \nabla w_{j,n,p}\textrm{d}x=o_j(1). \end{aligned}$$

That inequality together with (3.6) leads to

$$\begin{aligned} \begin{aligned}&\int _{{\mathbb {R}}^N}a|\nabla u_{j,n,p}|^{p-2}\nabla u_{j,n,p} \nabla w_{j,n,p} \textrm{d}x \\&\quad =o_j(1)+\int _{{\mathbb {R}}^N}a\left( |\nabla u_{j,n,p}|^{p-2}\nabla u_{j,n,p}-|\nabla u_{n,p}|^{p-2}\nabla u_{n,p}\right) \nabla w_{j,n,p}\textrm{d}x. \end{aligned} \end{aligned}$$
(3.7)

Similarly, we can show that

$$\begin{aligned} \begin{aligned}&\int _{{\mathbb {R}}^N}b| u_{j,n,p}|^{p-2} u_{j,n,p} w_{j,n,p} \textrm{d}x\\ {}&=o_j(1)+\int _{{\mathbb {R}}^N}b\left( | u_{j,n,p}|^{p-2}u_{j,n,p}-|u_{n,p}|^{p-2} u_{n,p}\right) (u_{j,n,p}-u_{n,p})\textrm{d}x. \end{aligned} \end{aligned}$$
(3.8)

On the other hand, since \(0<q<1,\) we have

$$\begin{aligned}{} & {} \int _{{\mathbb {R}}^N}h\left( u^+_{j,n,p}+\frac{1}{n} \right) ^{q-1} w_{j,n,p} \textrm{d}x\le \left( \frac{1}{n}\right) ^{q-1} \int _{{\mathbb {R}}^N}h |w_{j,n,p}| \textrm{d}x\\ {}{} & {} \quad =\left( \frac{1}{n}\right) ^{q-1} \int _{{\mathbb {R}}^N}\frac{h}{b^{\frac{1}{p}}}b^{\frac{1}{p}}|w_{j,n,p}|\textrm{d}x. \end{aligned}$$

Observing that \(b^{\frac{1}{p}}|w_{j,n,p}|\rightharpoonup 0\) weakly in the Lebesgue space \(L^{p}({\mathbb {R}}^N)\) and that the function \(\frac{h}{b^{\frac{1}{p}}}\) belongs to the space \(L^{\frac{p}{p-1}}({\mathbb {R}}^N)\) which is the dual of \(L^{p}({\mathbb {R}}^N), \) then

$$\begin{aligned} \int _{{\mathbb {R}}^N}\frac{h}{b^{\frac{1}{p}}}b^{\frac{1}{p}}|w_{j,n,p}| \textrm{d}x \rightarrow 0,\ \text{ as }\ j \rightarrow +\infty . \end{aligned}$$

It yields,

$$\begin{aligned} \int _{{\mathbb {R}}^N}h\left( u^+_{j,n,p}+\frac{1}{n} \right) ^{q-1} w_{j,n,p} \textrm{d}x =o_j(1). \end{aligned}$$
(3.9)

Combining (3.9), (3.8) and (3.7) with (3.5), we obtain

$$\begin{aligned} \begin{aligned}&\int _{{\mathbb {R}}^N}a\left( |\nabla u_{j,n,p}|^{p-2}\nabla u_{j,n,p}-|\nabla u_{n,p}|^{p-2}\nabla u_{n,p}\right) \nabla (u_{j,n,p}-u_{n,p})\textrm{d}x\\&\quad \quad + \int _{{\mathbb {R}}^N}b\left( | u_{j,n,p}|^{p-2}u_{j,n,p}-|u_{n,p}|^{p-2} u_{n,p}\right) (u_{j,n,p}-u_{n,p})\textrm{d}x\\&\quad =o_j(1). \end{aligned} \end{aligned}$$
(3.10)

Let us now recall the following elementary inequality [38, formula 2.2](see also [23, p. 713])

$$\begin{aligned} (p-1)|x-y|^p \le \Big [ \left( |x|^{p-2}x-|y|^{p-2}y\right) (x-y)\Big ]^\frac{p}{2}\left( |x|^p+|y|^p \right) ^\frac{2-p}{2}\ \text{ for }\ 1<p<2,\nonumber \\ \end{aligned}$$
(3.11)

for all \(x,y\in {\mathbb {R}}^N.\) Combining (3.10) and (3.11), we conclude that \(u_{j,n,p} \rightarrow u_{n,p}\) strongly in \(W^{1,p}_{a,b}({\mathbb {R}}^N). \) Therefore,

$$\begin{aligned} a^{\frac{1}{p}}(x) \nabla u_{j,n,p}(x) \rightarrow a^{\frac{1}{p}}(x) \nabla u_{n,p}(x)\ \text{ a.e. }\ x\in {\mathbb {R}}^N\ \text{ as }\ j \rightarrow +\infty . \end{aligned}$$

That last result allows us to use the Brezis–Lieb lemma to assert that

$$\begin{aligned} \int _{{\mathbb {R}}^N}a|\nabla u_{j,n,p} |^p \textrm{d}x =\int _{{\mathbb {R}}^N}a|\nabla w_{j,n,p} |^p \textrm{d}x+\int _{{\mathbb {R}}^N}a|\nabla u_{n,p} |^p \textrm{d}x+o_j(1). \qquad \end{aligned}$$
(3.12)

By (3.12) and (3.4), we get

$$\begin{aligned} \Vert u_{j,n,p}\Vert ^p_{W^{1,p}_{a,b}({\mathbb {R}}^N)}=\Vert w_{j,n,p}\Vert ^p_{W^{1,p}_{a,b}({\mathbb {R}}^N)}+\Vert u_{n,p}\Vert ^p_{W^{1,p}_{a,b}({\mathbb {R}}^N)}+o_j(1). \end{aligned}$$
(3.13)

Next, using the fact that \(\chi _n( u_{j,n,p})(x) \rightarrow \chi _n( u_{n,p})(x), \) a.e. \( x \in {\mathbb {R}}^N,\) together with the inequality

$$\begin{aligned} \Big | \chi _n(s)\Big |\le \left( \frac{1}{n} \right) ^{q-1}|s|,\ \forall s\in {\mathbb {R}}, \end{aligned}$$

we deduce that, up to a subsequence, \(b^{\frac{1}{p}}\chi _n( u_{j,n,p}) \rightharpoonup b^{\frac{1}{p}}\chi _n( u_{n,p})\) weakly in \(L^p({\mathbb {R}}^N)\) as \(j \rightarrow + \infty . \) Taking into account that \(h/b^{\frac{1}{p}}\in L^{\frac{p}{p-1}}({\mathbb {R}}^N),\) we infer

$$\begin{aligned} \int _{{\mathbb {R}}^N}h\chi _n( u_{j,n,p})\textrm{d}x=\int _{{\mathbb {R}}^N}\frac{h}{b^{\frac{1}{p}}}b^{\frac{1}{p}} \chi _n( u_{j,n,p})\textrm{d}x=\int _{{\mathbb {R}}^N}h\chi _n( u_{n,p})\textrm{d}x+o_j(1).\nonumber \\ \end{aligned}$$
(3.14)

Combining (3.13) and (3.14), we deduce that

$$\begin{aligned} m_{n,p}= & {} J_{n,p}(u_{j,n,p})+o_j(1)= \frac{1}{p}\Vert w_{j,n,p}\Vert _{W^{1,p}_{a,b}({\mathbb {R}}^N)}^p+\frac{1}{p}\Vert u_{n,p}\Vert _{W^{1,p}_{a,b}({\mathbb {R}}^N)}^p\nonumber \\{} & {} -\int _{{\mathbb {R}}^N}h \chi _n( u_{n,p})\textrm{d}x+o_j(1). \end{aligned}$$
(3.15)

It yields,

$$\begin{aligned} m_{n,p}\ge \frac{1}{p}\Vert u_{n,p}\Vert _{W^{1,p}_{a,b}({\mathbb {R}}^N)}^p-\int _{{\mathbb {R}}^N}h \chi _n( u_{n,p})\textrm{d}x+o_j(1), \end{aligned}$$

i.e.

$$\begin{aligned} m_{n,p}\ge J_{n,p}(u_{n,p})+o_j(1). \end{aligned}$$

Tending \(j\rightarrow +\infty ,\) it follows

$$\begin{aligned} m_{n,p}\ge J_{n,p}(u_{n,p}). \end{aligned}$$

Taking into account that \(\Vert u_{n,p}\Vert _{W^{1,p}_{a,b}({\mathbb {R}}^N)}\le \liminf \limits _{j \rightarrow + \infty } \Vert u_{j,n,p}\Vert _{W^{1,p}_{a,b}({\mathbb {R}}^N)} \le r,\) we deduce that \(m_{n,p}= J_{n,p}(u_{n,p}). \) Since \(m_{n,p}\le J_{n,p}(0)=0 \) and \(J_{n,p}(v)>0,\ \forall \ v \in W^{1,p}_{a,b}({\mathbb {R}}^N),\ \Vert v\Vert _{W^{1,p}_{a,b}({\mathbb {R}}^N)}=r, \) we conclude that \(\Vert u_{n,p}\Vert _{W^{1,p}_{a,b}({\mathbb {R}}^N)}< r. \) Having in mind that \(J_{n,p}\) is of class \(C^1,\) it follows that \(J'_{n,p}(u_{n,p})=0,\) i.e., \(u_{n,p}\) is a critical point of \(J_{n,p}.\) It is easy to see that \( u_{n,p} \ge 0. \) By the strong maximum principle (see [34, Theorem 2]), we can finally show that \(u_{n,p}(x)>0\) a.e. \( x \in {\mathbb {R}}^N. \) Since \((u_{n,p})_n \) is bounded in \( W^{1,p}_{a,b}({\mathbb {R}}^N), \) then there exists \( u_p \in W^{1,p}_{a,b}({\mathbb {R}}^N) \) such that, up to a subsequence, \(u_{n,p}\rightharpoonup u_p \) weakly in \( W^{1,p}_{a,b}({\mathbb {R}}^N),\ u_{n,p}(x) \rightarrow u_p(x) \) a.e. \( x\in {\mathbb {R}}^N. \) We claim that \( J_{n,p}(u_{n,p}) \rightarrow m_p,\ n \rightarrow +\infty . \) First, since \( J_{n,p}(u_{n,p})\ge J_p(u_{n,p}) \) and \(\Vert u_{n,p}\Vert _{W^{1,p}_{a,b}({\mathbb {R}}^N)}\le r, \) it yields

$$\begin{aligned} J_{n,p}(u_{n,p})\ge m_p \Rightarrow \liminf _{n \rightarrow + \infty } J_{n,p}(u_{n,p})\ge m_p. \end{aligned}$$
(3.16)

By the definition of \(m_p, \) we know that there exists a minimizing sequence \(\{y_k\}\) such that \(\lim \limits _{k \rightarrow +\infty } J_p(y_k)=m_p<0.\) we have

$$\begin{aligned} J_p(y_k)= & {} J_{n,p}(y_k)+ \int _{{\mathbb {R}}^N}h \chi _n( y_{k})\textrm{d}x-\frac{1}{q}\int _{{\mathbb {R}}^N}h ( y_{k}^+)^q\textrm{d}x\\\ge & {} m_{n,p}+ \int _{{\mathbb {R}}^N}h \chi _n( y_{k})\textrm{d}x-\frac{1}{q}\int _{{\mathbb {R}}^N}h ( y_{k}^+)^q\textrm{d}x. \end{aligned}$$

Using the same arguments as previously, we can easily show that

$$\begin{aligned}{} & {} \int _{{\mathbb {R}}^N}h \chi _n( y_{k})\textrm{d}x=\int _{{\mathbb {R}}^N}h \frac{\left( y_{k}^+ + \frac{1}{n}\right) ^q -\left( \frac{1}{n}\right) ^q}{q}\textrm{d}x +\left( \frac{1}{n}\right) ^{q-1}\int _{{\mathbb {R}}^N}h y_k^+ \textrm{d}x\\ {}{} & {} \quad \rightarrow \int _{{\mathbb {R}}^N}\frac{h \left( y_k^+\right) ^q}{q}\textrm{d}x,\ n \rightarrow +\infty . \end{aligned}$$

Thus,

$$\begin{aligned} J_p(y_k)\ge m_{n,p} +o_n(1). \end{aligned}$$

Tending n to \(+\infty , \) we obtain

$$\begin{aligned} J_p(y_k)\ge \limsup _{n \rightarrow +\infty }m_{n,p}=\limsup _{n \rightarrow +\infty } J_{n,p}(u_{n,p}). \end{aligned}$$

Finally, tending \(k \rightarrow + \infty ,\) we get

$$\begin{aligned} m_p\ge \limsup _{n \rightarrow +\infty } J_{n,p}(u_{n,p}). \end{aligned}$$
(3.17)

Combining (3.16) with (3.17), we deduce that \( \lim \limits _{n \rightarrow +\infty } J_{n,p}(u_{n,p})= m_p. \) Next, taking into account that \(J'_{n,p}(u_{n,p})=0, \) after introducing the sequence \(w_{n,p}=u_{n,p}-u_p,\) we can proceed as for the sequence \(\left( w_{j,n,p}\right) _j \) used at the beginning of the proof to prove that

$$\begin{aligned} m_p\ge & {} J_{n,p}(u_{n,p})+o_n(1)\\= & {} \frac{1}{p}\Vert w_{n,p}\Vert _{W^{1,p}_{a,b}({\mathbb {R}}^N)}^p+\frac{1}{p}\Vert u_p\Vert _{ W^{1,p}_{a,b}({\mathbb {R}}^N)}^p -\frac{1}{q}\int _{{\mathbb {R}}^N} h \left( u_p^+\right) ^q \textrm{d}x+o_n(1)\\\ge & {} J_p(u_p)+o_n(1). \end{aligned}$$

Tending \(n \rightarrow + \infty , \) we deduce that \(m_p\ge J_p(u_p).\) Hence, \(J_p(u_p)= m_p< 0. \) It follows that \(u_p\) is a local minimizer of \(J_p. \) The next step in the proof consists of proving that \(u_p\) is a solution of (3.1). Since \(u_p\) is a local minimizer of \(J_p, \) then for any \(\psi \in W^{1,p}_{a,b}({\mathbb {R}}^N),\ \psi \ge 0, \) and for all \( 0<t<1 \) small enough such that \(\Vert u_p +t \psi \Vert _{ W^{1,p}_{a,b}({\mathbb {R}}^N)}\le r,\) one has

$$\begin{aligned} 0\le & {} J_p(u_p+t \psi )-J_p(u_p)= \frac{\Vert u_p+t \psi \Vert ^p_{ W^{1,p}_{a,b}({\mathbb {R}}^N)}-\Vert u_p\Vert ^p_{ W^{1,p}_{a,b}({\mathbb {R}}^N)}}{p}\\{} & {} -\frac{1}{q}\int _{{\mathbb {R}}^N}h\left( ((u_p+t \psi ))^q-(u_p )^q\right) \textrm{d}x. \end{aligned}$$

Dividing by \(t>0 \) and passing to the limit as \(t \rightarrow 0^+,\) it gives

$$\begin{aligned} \begin{aligned} \frac{1}{q}\liminf _{t \rightarrow 0^+}\int _{{\mathbb {R}}^N}\frac{h\left( ((u_p+t \psi ))^q-(u_p)^q\right) }{t}\textrm{d}x&\le \liminf _{t \rightarrow 0^+} \left( \frac{\Vert u_p+t \psi \Vert ^p_{ W^{1,p}_{a,b}({\mathbb {R}}^N)}-\Vert u_p\Vert ^p_{ W^{1,p}_{a,b}({\mathbb {R}}^N)}}{pt}\right) \\&= \int _{{\mathbb {R}}^N}a(x)|\nabla u_p|^{p-2}\nabla u_p.\nabla \psi \textrm{d}x+\int _{{\mathbb {R}}^N}b(x)|u_p|^{p-2} u_p \psi \textrm{d}x. \end{aligned} \end{aligned}$$
(3.18)

Since \(\left( u_p+t \psi \right) \ge u_p. \) Thus, by using Fatou’s lemma, we have

$$\begin{aligned} \int _{{\mathbb {R}}^N}\frac{h \psi }{u_p^{1-q}}\textrm{d}x \le \frac{1}{q}\liminf _{t \rightarrow 0^+}\int _{{\mathbb {R}}^N}\frac{h\left( ((u_p+t \psi ))^q-(u_p)^q\right) }{t} \textrm{d}x. \end{aligned}$$

Putting that last inequality in (3.18), we infer

$$\begin{aligned} \int _{{\mathbb {R}}^N}\frac{h \psi }{u_p^{1-q}}\textrm{d}x\le & {} \int _{{\mathbb {R}}^N}a|\nabla u_p|^{p-2}\nabla u_p.\nabla \psi \textrm{d}x \nonumber \\ {}{} & {} +\int _{{\mathbb {R}}^N}b|u_p|^{p-2} u_p \psi \textrm{d}x,\ \forall \ 0\le \psi \in W^{1,p}_{a,b}({\mathbb {R}}^N). \end{aligned}$$
(3.19)

Since \(\Vert u_p\Vert _{ W^{1,p}_{a,b}({\mathbb {R}}^N)}< r,\) then \(\Vert u_p(1+t )\Vert _{ W^{1,p}_{a,b}({\mathbb {R}}^N)}< r,\ \forall \ -1<t<\frac{r}{\Vert u_p\Vert _{ W^{1,p}_{a,b}({\mathbb {R}}^N)}}-1.\) We define the function \(k:]-1,-1+\frac{r}{\Vert u_p\Vert _{ W^{1,p}_{a,b}({\mathbb {R}}^N)}}[ \rightarrow {\mathbb {R}} \) by \(k(t)=J_p((1+t)u_p).\) Clearly, k attains its minimum at \(t=0.\) Since k is derivable at \(t=0,\) then

$$\begin{aligned} k'(0)=0\Leftrightarrow \Vert u_p\Vert _{ W^{1,p}_{a,b}({\mathbb {R}}^N)}^p-\int _{{\mathbb {R}}^N}h(u_p)^q\textrm{d}x=0. \end{aligned}$$
(3.20)

Now, let \(\varphi \in W^{1,p}_{a,b}({\mathbb {R}}^N)\) and \(\epsilon >0\). Since \((u_p +\epsilon \varphi )^+\in W^{1,p}_{a,b}({\mathbb {R}}^N)\) and \((u_p +\epsilon \varphi )^+\ge 0,\) then by (3.19) we deduce that

$$\begin{aligned} \begin{aligned} 0&\le \int _{{\mathbb {R}}^N}a|\nabla u_p|^{p-2}\nabla u_p.\nabla (u_p +\epsilon \varphi )^+ \textrm{d}x+\int _{{\mathbb {R}}^N}b|u_p|^{p-2} u_p (u_p +\epsilon \varphi )^+ \textrm{d}x\\ {}&\quad - \int _{{\mathbb {R}}^N}\frac{h (u_p +\epsilon \varphi )^+}{u_p^{1-q}}\textrm{d}x\\&=\int _{{\mathbb {R}}^N}a|\nabla u_p|^{p-2}\nabla u_p.\nabla (u_p +\epsilon \varphi ) \textrm{d}x+\int _{{\mathbb {R}}^N}b|u_p|^{p-2} u_p (u_p+\epsilon \varphi ) \textrm{d}x\\&\quad - \int _{{\mathbb {R}}^N}\frac{h (u_p +\epsilon \varphi )}{u_p^{1-q}}\textrm{d}x-\int _{u_p+ \epsilon \varphi<0}a|\nabla u_p|^{p-2}\nabla u_p.\nabla (u_p+\epsilon \varphi ) \textrm{d}x\\&\quad -\int _{u_p+ \epsilon \varphi<0}b|u_p|^{p-2} u_p (u_p+\epsilon \varphi ) \textrm{d}x + \int _{u_p+ \epsilon \varphi<0}\frac{h (u_p+\epsilon \varphi )}{u_p^{1-q}}\textrm{d}x\\&=\epsilon \left( \int _{{\mathbb {R}}^N}a|\nabla u_p|^{p-2}\nabla u_p.\nabla \varphi \textrm{d}x+\int _{{\mathbb {R}}^N}b|u_p|^{p-2} u_p \varphi \textrm{d}x\right) \\&\quad -\epsilon \int _{{\mathbb {R}}^N}\frac{h \varphi }{u_p^{1-q}}\textrm{d}x+\Vert u_p\Vert _{W^{1,p}_{a,b}({\mathbb {R}}^N)}^p-\int _{{\mathbb {R}}^N}h(u_p)^q\textrm{d}x\\&\quad - \int _{u_p+ \epsilon \varphi<0}a|\nabla u_p|^{p-2}\nabla u_p.\nabla (u_p+\epsilon \varphi ) \textrm{d}x\\&\quad -\int _{u_p+ \epsilon \varphi<0}b|u_p|^{p-2} u_p (u_p+\epsilon \varphi ) \textrm{d}x+ \int _{u_p+ \epsilon \varphi<0}\frac{h (u_p+\epsilon \varphi )}{u_p^{1-q}}\textrm{d}x\\&\le \epsilon \left( \int _{{\mathbb {R}}^N}a|\nabla u_p|^{p-2}\nabla u_p.\nabla \varphi \textrm{d}x+\int _{{\mathbb {R}}^N}b|u_p|^{p-2} u_p \varphi \textrm{d}x\right) \\&\quad -\epsilon \int _{{\mathbb {R}}^N}\frac{h \varphi }{u_p^{1-q}}\textrm{d}x+\Vert u_p\Vert _{W^{1,p}_{a,b}({\mathbb {R}}^N)}^p-\int _{{\mathbb {R}}^N}h(u_p)^q\textrm{d}x\\&\quad -\epsilon \left( \int _{u_p+ \epsilon \varphi<0}a|\nabla u_p|^{p-2}\nabla u_p.\nabla \varphi \textrm{d}x -\int _{u_p+ \epsilon \varphi <0}b|u_p|^{p-2} u_p \varphi \textrm{d}x\right) . \end{aligned} \end{aligned}$$
(3.21)

Using (3.20) and dividing by \(\epsilon >0 \) in (3.21), it follows

$$\begin{aligned} \begin{aligned}&0\le \int _{{\mathbb {R}}^N}a|\nabla u_p|^{p-2}\nabla u_p.\nabla \varphi \textrm{d}x+\int _{{\mathbb {R}}^N}b|u_p|^{p-2} u_p \varphi \textrm{d}x - \int _{{\mathbb {R}}^N}\frac{h \varphi }{u_p^{1-q}}\textrm{d}x\\&\quad \quad - \left( \int _{u_p+ \epsilon \varphi<0}a|\nabla u_p|^{p-2}\nabla u_p.\nabla \varphi \textrm{d}x -\int _{u_p+ \epsilon \varphi <0}b|u_p|^{p-2} u_p \varphi \textrm{d}x\right) . \end{aligned} \end{aligned}$$
(3.22)

Since \(1_{\{u_p ^+ +\epsilon \varphi < 0\}}(x) \rightarrow 0, \) a.e. \( x\in {\mathbb {R}}^N,\) as \( \epsilon \rightarrow 0^+, \) then we can use the Lebesgue-dominated convergence theorem to deduce that all the integrals over the set \(\{u_p ^+ +\epsilon \varphi <0\}\) appearing in (3.22) tend to zero as \(\epsilon \rightarrow 0^+. \) Hence,

$$\begin{aligned} 0\le \int _{{\mathbb {R}}^N}a|\nabla u_p|^{p-2}\nabla u_p.\nabla \varphi \textrm{d}x+\int _{{\mathbb {R}}^N}b|u_p|^{p-2} u_p \varphi \textrm{d}x - \int _{{\mathbb {R}}^N}\frac{h \varphi }{u_p^{1-q}}\textrm{d}x. \end{aligned}$$

By the arbitrariness of \(\varphi , \) this inequality also holds for \(-\varphi , \) i.e.

$$\begin{aligned} \int _{{\mathbb {R}}^N}a|\nabla u_p|^{p-2}\nabla u_p.\nabla \varphi \textrm{d}x+\int _{{\mathbb {R}}^N}b|u_p|^{p-2} u_p \varphi \textrm{d}x =\int _{{\mathbb {R}}^N}\frac{h \varphi }{u_p^{1-q}}\textrm{d}x,\ \forall \varphi \in W^{1,p}_{a,b}({\mathbb {R}}^N).\nonumber \\ \end{aligned}$$
(3.23)

The strong maximum principle implies that \( u_p(x) > 0, \) a.e. \( x \in {\mathbb {R}}^N. \) \(\square \)

Now, let us show that \(u_p\in L^{\infty }({\mathbb {R}}^N).\) More precisely, we prove the following result.

Lemma 6

Let \(u_p\) be a solution of (3.1). Then, there exists \(k>0\) independent of p such that

$$\begin{aligned} \left| u_p\right| _{L^{\infty }({\mathbb {R}}^N)}\le k. \end{aligned}$$

Proof

Taking \(\left( u_p- k\right) ^+\) with \(k\in ]1, +\infty [\) as test function in (3.1), it yields

$$\begin{aligned} \int _{{\mathbb {R}}^N}a|\nabla u_p|^{p-2} \nabla u_p \nabla \left( u_p- k\right) ^+\textrm{d}x+\int _{{\mathbb {R}}^N}bu_p^{p-1}\left( u_p- k\right) ^+\textrm{d}x= \int _{{\mathbb {R}}^N}\frac{h}{u_p^{1-q}}\left( u_p- k\right) ^+ \textrm{d}x. \end{aligned}$$

Since

$$\begin{aligned} \int _{{\mathbb {R}}^N}a|\nabla u_p|^{p-2} \nabla u_p \nabla \left( u_p- k\right) ^+\textrm{d}x = \int _{{\mathbb {R}}^N} a \left| \nabla (u_p -k)^+\right| ^p \textrm{d}x \ge 0, \end{aligned}$$

we infer

$$\begin{aligned} \int _{{\mathbb {R}}^N}bu_p^{p-1}\left( u_p- k\right) ^+\textrm{d}x\le & {} \int _{{\mathbb {R}}^N}\frac{h}{u_p^{1-q}}\left( u_p- k\right) ^+ \textrm{d}x\\\le & {} \frac{1}{k^{1-q}}\int _{{\mathbb {R}}^N}h\frac{b}{b}\left( u_p- k\right) ^+ \textrm{d}x\\\le & {} \frac{k^{1-p}}{k^{1-q}}\left| \frac{h}{b}\right| _{L^{\infty }({\mathbb {R}}^N)}\int _{{\mathbb {R}}^N}bk^{p-1}\left( u_p- k\right) ^+ \textrm{d}x\\\le & {} k^{q-p}\left| \frac{h}{b}\right| _{L^{\infty }({\mathbb {R}}^N)}\int _{{\mathbb {R}}^N}bk^{p-1}\left( u_p- k\right) ^+ \textrm{d}x. \end{aligned}$$

Now, choosing \(k>1\) such that \(\left| \frac{h}{b}\right| _{L^{\infty }({\mathbb {R}}^N)}\le k^{1-q}\le k^{p-q},\ \forall \ 1<p<2,\) we deduce that

$$\begin{aligned} \int _{{\mathbb {R}}^N}b\left( u_p^{p-1}-k^{p-1}\right) \left( u_p- k\right) ^+\textrm{d}x\le 0. \end{aligned}$$

Therefore, \((u_p-k)^+=0\) and by consequence \(u_p(x)\le k,\) a.e. \(x \in {\mathbb {R}}^N.\) \(\square \)

To conclude the proof of Theorem 3.1, it remains to show that

$$\begin{aligned} \int _{{\mathbb {R}}^N}\frac{h \varphi }{u_p^{1-q}}\textrm{d}x < + \infty ,\ \forall \varphi \in W^{1,p}_{a,b}({\mathbb {R}}^N). \end{aligned}$$

For that aim, it suffices to use Young’s inequality and (3.23) to obtain

$$\begin{aligned} \int _{{\mathbb {R}}^N}\frac{h \varphi }{u_p ^{1-q}}\textrm{d}x= & {} \int _{{\mathbb {R}}^N}a|\nabla u_p|^{p-2}\nabla u_p.\nabla \varphi \textrm{d}x+\int _{{\mathbb {R}}^N}b u_p^{p-1} \varphi \textrm{d}x\\\le & {} \int _{{\mathbb {R}}^N}a|\nabla u_p|^p \textrm{d}x+\int _{{\mathbb {R}}^N}bu_p^p\textrm{d}x+\int _{{\mathbb {R}}^N}a|\nabla \varphi |^p\textrm{d}x +\int _{{\mathbb {R}}^N}b|\varphi |^p \textrm{d}x\\\le & {} \Vert u_p\Vert ^p_{W^{1,p}_{a,b}({\mathbb {R}}^N)}+\Vert \varphi \Vert ^p_{W^{1,p}_{a,b}({\mathbb {R}}^N)}< +\infty ,\ \forall \varphi \in W^{1,p}_{a,b}({\mathbb {R}}^N). \end{aligned}$$

4 Completion of the Proof of Theorem 1.1: The Limit Problem

First, observe that the family \((u_p)_{1<p<2}\) is bounded in \(\textrm{BV}^{b}_{a}({\mathbb {R}}^N).\) Indeed, taking \(u_p\) as test function in problem (3.1) and by using the Hölder inequality and Lemma 2, we get

$$\begin{aligned} \int _{{\mathbb {R}}^N}a|\nabla u_{p}|^{p}\ \textrm{d}x+\int _{{\mathbb {R}}^N}bu_{p}^{p} \textrm{d}x= & {} \int _{{\mathbb {R}}^N}\frac{h}{u_p^{1-q}}u_p\textrm{d}x\\\le & {} \int _{{\mathbb {R}}^N}hu_p^{q}\textrm{d}x\\\le & {} \int _{{\mathbb {R}}^N}\frac{h}{b^q} b^qu_p^{q}\textrm{d}x\\\le & {} \left( \int _{{\mathbb {R}}^N}\left( \frac{h}{b^q}\right) ^\frac{1}{1-q}\textrm{d}x\right) ^{1-q} \left( \int _{{\mathbb {R}}^N}bu_p\textrm{d}x\right) ^q\\\le & {} \left( \int _{{\mathbb {R}}^N}\left( \frac{h}{b^q}\right) ^\frac{1}{1-q}\textrm{d}x\right) ^{1-q} \Vert u_p\Vert ^q_{W^{1,1}_{a,b}({\mathbb {R}}^N)}\\\le & {} |\frac{h}{b^q}|_{L^{\frac{1}{1-q}}({\mathbb {R}}^N)}C\left\| u_p\right\| ^q_{W^{1,p}_{a,b}({\mathbb {R}}^N)}\\\le & {} C_0\left\| u_p\right\| ^q_{W^{1,p}_{a,b}({\mathbb {R}}^N)}. \end{aligned}$$

Thus,

$$\begin{aligned} \left\| u_p\right\| ^{p}_{W^{1,p}_{a,b}({\mathbb {R}}^N)}\le C_0^{\frac{p}{p-q}}\le C_0^{\frac{2}{1-q}}\le {\overline{C}}. \end{aligned}$$
(4.1)

The Young’s inequality together with (4.1) implies

$$\begin{aligned} \begin{aligned} \Vert u_{p}\Vert _{W^{1,1}_{a,b}({\mathbb {R}}^N)}&=\int _{{\mathbb {R}}^N}a|\nabla u_{p}|\textrm{d}x+ \int _{{\mathbb {R}}^N}b|u_{p}|\textrm{d}x\\&\le \frac{1}{p}\Big ( \int _{{\mathbb {R}}^N}a|\nabla u_{p}|^p\textrm{d}x+\int _{{\mathbb {R}}^N}bu_{p}^p\textrm{d}x \Big )+\frac{p-1}{p}\Big (\int _{{\mathbb {R}}^N}a\textrm{d}x+\int _{{\mathbb {R}}^N}b\textrm{d}x\Big )\\&\le \Vert u_p\Vert _{W^{1,p}_{a,b}({\mathbb {R}}^N)}^p+|a|_{L^1({\mathbb {R}}^N)}+|b|_{L^1({\mathbb {R}}^N)}\\&\le {\overline{C}}+|a|_{L^1({\mathbb {R}}^N)}+|b|_{L^1({\mathbb {R}}^N)}\\&\le {\hat{C}}, \end{aligned} \end{aligned}$$
(4.2)

where \({\hat{C}}\) is a positive constant independent of p. Having in mind that \(W^{1,1}_{a,b}({\mathbb {R}}^N) \) is continuously embedded into \( \textrm{BV}_{a}^{b}({\mathbb {R}}^N), \) then, from (4.2), we can assert that the family \( (u_p)_{1< p < 2} \) is bounded in \( \textrm{BV}_a^b( {\mathbb {R}}^N). \)

Hence, by the virtue of the local compact embeddings of \( \textrm{BV}_a^b( {\mathbb {R}}^N), \) we deduce the existence of a function \(u\in \textrm{BV}_{a}^{b}({\mathbb {R}}^N)\) such that

$$\begin{aligned} u_{p}(x)\rightarrow u(x),\ \text{ a.e. }\ x\in {\mathbb {R}}^N,\\ a\nabla u_{p}\rightharpoonup a|Du|\ \text{ weakly }\,\, ^*\,\, \text {as measures},\\ u_{p}\rightarrow u \ \text{ in }\ L^t_{\textrm{loc}}( {\mathbb {R}}^N),\ \text{ for } \text{ all }\ t\in [1,1^*), \end{aligned}$$

for all \( 1\le t< 1^* \) and B ball of \( {\mathbb {R}}^N, \) there exists \( g_t\in L^t( B) \)such that

$$\begin{aligned} \left| u_p(x)\right| \le g_t(x),\ \text{ a.e. }\ x \in B. \end{aligned}$$

Observe that, from Lemma 6, we immediately see that \(u\in L^{\infty }({\mathbb {R}}^N).\)

Now let us prove that \(\frac{h}{u^{1-q}}\in L^1_{\textrm{loc}}({\mathbb {R}}^N).\) For that aim, fix \(1<p <2 \) and let \(\varphi \in C^{\infty }_c({\mathbb {R}}^N).\) Applying Young’s inequality, by (3.1) and (4.1), it yields

$$\begin{aligned} \begin{aligned} \int _{{\mathbb {R}}^N}\frac{h}{u_p^{1-q}}\varphi \textrm{d}x&=\int _{{\mathbb {R}}^N}a|\nabla u_p|^{p-2} \nabla u_p.\nabla \varphi \textrm{d}x+\int _{{\mathbb {R}}^N}bu_p^{p-1} \varphi \textrm{d}x\\&\le \frac{p-1}{p} \Vert u_p\Vert ^p_{W^{1,p}_{a,b}({\mathbb {R}}^N)} +\frac{1}{p}\left[ \int _{{\mathbb {R}}^N}a\left| \nabla \varphi \right| ^p\textrm{d}x +\int _{{\mathbb {R}}^N}b\left| \varphi \right| ^p\textrm{d}x \right] \\&\le {\overline{C}} + |a|_{L^1({\mathbb {R}}^N)}+|b|_{L^1({\mathbb {R}}^N)}+\int _{{\mathbb {R}}^N}a\left| \nabla \varphi \right| ^2 \textrm{d}x+\int _{{\mathbb {R}}^N}b\varphi ^2 \textrm{d}x. \end{aligned} \end{aligned}$$
(4.3)

Fatou’s lemma gives

$$\begin{aligned} \begin{aligned} \int _{{\mathbb {R}}^N}\frac{h}{u^{1-q}}\varphi \textrm{d}x&\le \liminf _{p\rightarrow 1^+} \int _{{\mathbb {R}}^N}\frac{h}{u_p^{1-q}}\varphi \textrm{d}x\\&\le C + \int _{{\mathbb {R}}^N}a\left| \nabla \varphi \right| ^2 \textrm{d}x+\int _{{\mathbb {R}}^N}b\varphi ^2 \textrm{d}x,\ \forall \ \varphi \in C^{\infty }_c({\mathbb {R}}^N),\ \varphi \ge 0. \end{aligned} \end{aligned}$$
(4.4)

This implies that \(\frac{h}{u^{1-q}}\in L^1_{\textrm{loc}}({\mathbb {R}}^N). \)

Remark 3

Let us point out that since

$$\begin{aligned} \int _{{\mathbb {R}}^N} \frac{h}{u^{1-q}} \varphi \textrm{d}x < \infty ,\ \forall \varphi \in C^\infty _c({\mathbb {R}}^N),\ \varphi \ge 0, \end{aligned}$$

then

$$\begin{aligned} |\{x \in {\mathbb {R}}^N:\ u(x)=0,\ h(x)>0\}|=0, \end{aligned}$$

which means that

$$\begin{aligned} \{x \in {\mathbb {R}}^N:\ u(x)=0\}\subseteq \{ x \in {\mathbb {R}}^N:\ h(x)=0\}. \end{aligned}$$

Lemma 7

There exist \(z\in L^{\infty }({\mathbb {R}}^N,{\mathbb {R}}^N)\) and \(\gamma \in L^{\infty }({\mathbb {R}}^N)\) such that

$$\begin{aligned} |z|_{L^{\infty }( {\mathbb {R}}^N, {\mathbb {R}}^N)}\le 1,\ -\text{ div }(\textrm{az}) \in {\mathcal {M}}(B),\ \text{ for } \text{ all } \text{ balls }\ B\subset {\mathbb {R}}^N, \end{aligned}$$
(4.5)
$$\begin{aligned} \left| \gamma \right| _{L^{\infty }( {\mathbb {R}}^N)}\le 1,\ \gamma u=|u|,\ \text{ a.e. } \text{ in }\ {\mathbb {R}}^N, \end{aligned}$$
(4.6)

and, up to subsequences, the following convergences hold as \( p \rightarrow 1^+, \)

$$\begin{aligned} a|\nabla u_{p}|^{p-2}\nabla u_{p}\rightharpoonup a z\ \text{ weakly } \text{ in }\ L^{1}(B, {\mathbb {R}}^N), \end{aligned}$$
(4.7)
$$\begin{aligned} b u_{p}^{p-1}\rightharpoonup b \gamma \ \text{ weakly } \text{ in }\ L^{1}(B), \end{aligned}$$
(4.8)

for all balls \(B\subset {\mathbb {R}}^N.\)

Proof

Let \(B=B(0, R)\) be a ball of \({\mathbb {R}}^N.\) Fix \(r\in (1,+\infty )\) and consider \(1< p < \frac{r}{r-1}.\) Using the continuous embedding \(W^{1,p}_{a,b}({\mathbb {R}}^N)\hookrightarrow W^{1,p}(B), \) it follows that

$$\begin{aligned} \begin{aligned} \int _{B}|\nabla u_{p}|^{(p-1)r}\textrm{d}x&\le \left( \int _{B}|\nabla u_{p}|^{p}\textrm{d}x\right) ^{\frac{r(p-1)}{p}}|B|^{1-\frac{r(p-1)}{p}}\\&\le C_{R}^{\frac{r(p-1)}{p}}|B|^{1-\frac{r(p-1)}{p}}, \end{aligned} \end{aligned}$$
(4.9)

where \( C_R \) is a positive constant depending only on R. Thus, up to a subsequence, there exists \(z_r\in L^{r}(B, {\mathbb {R}}^N)\) such that

$$\begin{aligned} |\nabla u_{p}|^{p-2}\nabla u_{p}\rightharpoonup z_r\ \text{ weakly } \text{ in } \ L^{r}(B, {\mathbb {R}}^N),\ \text{ as }\ p \rightarrow 1^+. \end{aligned}$$

Now, through a standard diagonal argument, one can deduce the existence of a unique vector field z defined independently of r,  such that

$$\begin{aligned} |\nabla u_{p}|^{p-2}\nabla u_{p}\rightharpoonup z\ \text{ weakly } \text{ in } \ L^{r}(B, {\mathbb {R}}^N),\ \forall \ r > 1. \end{aligned}$$
(4.10)

Moreover, letting \(p\rightarrow 1^+,\) one yields to

$$\begin{aligned} |z|_{L^{r}(B, {\mathbb {R}}^N)}\le |B|^{\frac{1}{r}},\ \forall \ 1<r< \infty . \end{aligned}$$

Letting \(r\rightarrow +\infty ,\) we also have \( z \in L^{\infty }(B, {\mathbb {R}}^N) \) and \( |z|_{L^{\infty }(B, {\mathbb {R}}^N)}\le 1.\) Since B is arbitrary, we can finally conclude that

$$\begin{aligned} z\in L^{\infty }({\mathbb {R}}^N, {\mathbb {R}}^N),\ \text{ and }\ |z|_{L^{\infty }( {\mathbb {R}}^N, {\mathbb {R}}^N)}\le 1. \end{aligned}$$
(4.11)

On the other hand, observe that \( |\nabla u_{p}|^{p-2}\nabla u_{p}\rightharpoonup z\) weakly in \(L^{1}(B, {\mathbb {R}}^N).\) Since a is bounded from below on B,  then we get

$$\begin{aligned} a|\nabla u_{p}|^{p-2}\nabla u_{p}\rightharpoonup a z,\ \text{ weakly } \text{ in } \ L^1(B, {\mathbb {R}}^N)\ \text{ as }\ \ p \rightarrow 1^+. \end{aligned}$$
(4.12)

Hence,

$$\begin{aligned} \int _{{\mathbb {R}}^N} a|\nabla u_{p}|^{p-2}\nabla u_{p}.\nabla \varphi \textrm{d}x \rightarrow \int _{{\mathbb {R}}^N} a z.\nabla \varphi \ \textrm{d}x,\ \forall \ \varphi \in C_c^{\infty }( {\mathbb {R}}^N). \end{aligned}$$
(4.13)

Similarly, we can also show that there exists \( \gamma \in L^{\infty }( {\mathbb {R}}^N) \) such that \( \left| \gamma \right| _{L^{\infty }( {\mathbb {R}}^N)} \le 1 \) and

$$\begin{aligned} u_p^{p-1} \rightharpoonup \gamma \ \text{ weakly } \text{ in }\ L^1(B)\ \text{ as }\ p \rightarrow 1^+. \end{aligned}$$
(4.14)

Moreover, we know that, if \(u(x)\ne 0,\) then we have

$$\begin{aligned} (u_p(x))^{p-1}\rightarrow \frac{u(x)}{|u(x)|},\ \text{ a.e. }\ x\in {\mathbb {R}}^N. \end{aligned}$$
(4.15)

Combining (4.14) and (4.15), we deduce that

$$\begin{aligned} \gamma u=|u|,\ \text{ a.e. }\ \text{ in }\ {\mathbb {R}}^N. \end{aligned}$$

Now, using the Lebesgue-dominated convergence theorem, we can easily see that

$$\begin{aligned} \int _{{\mathbb {R}}^N}b u_p^{p-1} \varphi \ \textrm{d}x \rightarrow \int _{{\mathbb {R}}^N}b\gamma \ \varphi \ \textrm{d}x,\ \forall \ \varphi \in C_c^{\infty }( {\mathbb {R}}^N). \end{aligned}$$
(4.16)

It remains to verify that \(\text{ div }\left( a z\right) \in {\mathcal {M}}\left( B\right) ,\) for all balls \(B\subset {\mathbb {R}}^N. \) To do this, we take \(0\le \varphi \in C^{\infty }_c({\mathbb {R}}^N)\) with \( \text{ supp } (\varphi )\subset B \) as test function in (3.1) to obtain

$$\begin{aligned} \int _{{\mathbb {R}}^N}\frac{h}{u_p^{1-q}}\varphi \textrm{d}x= & {} \int _{{\mathbb {R}}^N}a|\nabla u_p|^{p-2} \nabla u_p.\nabla \varphi \textrm{d}x+\int _{{\mathbb {R}}^N}b u_p^{p-1} \varphi \textrm{d}x. \end{aligned}$$
(4.17)

By (4.3), (4.13) and (4.17), we infer

$$\begin{aligned} -\int _{{\mathbb {R}}^N}\varphi \ \text{ div }\left( a z\right) = -\left\langle \text{ div } (a z), \varphi \right\rangle = \int _{{\mathbb {R}}^N}a z .\nabla \varphi \textrm{d}x \le \lim _{p \rightarrow 1^+}\int _{{\mathbb {R}}^N}\frac{h}{u_p^{1-q}} \varphi \textrm{d}x<+ \infty . \end{aligned}$$

By the arbitrariness of \(\varphi \in C^{\infty }_c({\mathbb {R}}^N),\ \varphi \ge 0, \) we deduce that the total variation of \(-\text{ div }\left( a z\right) \) is locally finite, that is \( \text{ div }(a z) \in {\mathcal {M}}(B),\ \forall \ B \) balls of \( {\mathbb {R}}^N. \) \(\square \)

Combining (4.13), (4.16) and using Fatou’s lemma in (4.17), it follows that

$$\begin{aligned} \int _{{\mathbb {R}}^N}\textrm{a z}. \nabla \varphi \textrm{d}x + \int _{{\mathbb {R}}^N}b \gamma \varphi \textrm{d}x \ge \int _{{\mathbb {R}}^N} \frac{h}{u^{1-q}} \varphi \textrm{d}x,\ \forall \varphi \in C^{\infty }_c({\mathbb {R}}^N),\ \varphi \ge 0. \end{aligned}$$

Therefore,

$$\begin{aligned} - \text{ div }\ (\textrm{a z}) + b \gamma \ge \frac{h}{u^{1-q}},\ \text{ in }\ \ D'({\mathbb {R}}^N). \end{aligned}$$
(4.18)

Now, we claim that

$$\begin{aligned} (\textrm{az},\mathrm{{Du}})=a|\mathrm{{Du}}|,\ \text{ as } \text{ measures } \text{ in }\ {\mathbb {R}}^N. \end{aligned}$$
(4.19)

Lemma 8

The vector field z satisfies (4.19).

Proof

Let B be a ball of \({\mathbb {R}}^N.\) Let \(0 \le \varphi \in C^{\infty }_{c}({\mathbb {R}}^N)\) be such that \( \text{ supp }( \varphi ) \subset B.\) If we consider \(u_{p}\varphi \in W^{1,p}_{a,b}({\mathbb {R}}^N)\) as test function in (3.1), we obtain

$$\begin{aligned} \int _{{\mathbb {R}}^N}a|\nabla u_{p}|^p\varphi \ \textrm{d}x +\int _{{\mathbb {R}}^N}au_{p}|\nabla u_{p}|^{p-2}\nabla u_{p}.\nabla \varphi \ \textrm{d}x+\int _{{\mathbb {R}}^N}b| u_{p}|^p\varphi \ \textrm{d}x=\int _{{\mathbb {R}}^N}h u_p^q\varphi \ \textrm{d}x. \end{aligned}$$

Applying Young’s inequality, it yields

$$\begin{aligned} \begin{aligned} \int _{{\mathbb {R}}^N}\varphi a|\nabla u_{p}| \ \textrm{d}x&+\int _{{\mathbb {R}}^N}au_{p}|\nabla u_{p}|^{p-2}\nabla u_{p}.\nabla \varphi \ \textrm{d}x+\int _{{\mathbb {R}}^N}b| u_{p}|^p\varphi \ \textrm{d}x\\&\le \int _{{\mathbb {R}}^N}a|\nabla u_{p}|^p\varphi \ \textrm{d}x+\frac{p-1}{p}\int _{{\mathbb {R}}^N}a\varphi \ \textrm{d}x\\&\quad +\int _{{\mathbb {R}}^N}au_{p}|\nabla u_{p}|^{p-2}\nabla u_{p}.\nabla \varphi \ \textrm{d}x+\int _{{\mathbb {R}}^N}b| u_{p}|^p\varphi \ \textrm{d}x\\&\le \int _{{\mathbb {R}}^N}hu_{p}^q\varphi \ \textrm{d}x +\frac{p-1}{p}\int _{{\mathbb {R}}^N}a\varphi \ \textrm{d}x. \end{aligned} \end{aligned}$$
(4.20)

Now, using again Young’s inequality and taking the lower semicontinuity of the map \(u\longmapsto \int _{{\mathbb {R}}^N}\varphi a(x)|\mathrm{{Du}}|\) with respect to the \(L^1 ({\mathbb {R}}^N)\) convergence into account, we obtain

$$\begin{aligned} \begin{aligned} \int _{{\mathbb {R}}^N}\varphi a|\mathrm{{Du}}|&\le \liminf _{p\rightarrow 1^+}\int _{{\mathbb {R}}^N}\varphi a|\nabla u_{p}| \ \textrm{d}x\\&\le \liminf _{p\rightarrow 1^+}\left( \frac{1}{p}\int _{{\mathbb {R}}^N}\varphi a|\nabla u_{p}|^p \ \textrm{d}x +\frac{p-1}{p}\int _{{\mathbb {R}}^N}a\varphi \ \textrm{d}x \right) \\&\le \liminf _{p\rightarrow 1^+}\int _{{\mathbb {R}}^N}\varphi a|\nabla u_{p}|^p \textrm{d}x. \end{aligned} \end{aligned}$$
(4.21)

Moreover, by (4.7) and (4.8), it follows that

$$\begin{aligned} \int _{{\mathbb {R}}^N}au_{p}|\nabla u_{p}|^{p-2}\nabla u_{p}.\nabla \varphi \ \textrm{d}x \rightarrow \int _{{\mathbb {R}}^N}ua z \cdot \nabla \varphi \ \textrm{d}x \end{aligned}$$
(4.22)

and

$$\begin{aligned} \int _{{\mathbb {R}}^N}bu_{p}^{p} \varphi \ \textrm{d}x\rightarrow \int _{{\mathbb {R}}^N} b \gamma \ u \varphi \ \textrm{d}x. \end{aligned}$$
(4.23)

Finally, we claim that

$$\begin{aligned} \lim _{p\rightarrow 1^+}\int _{{\mathbb {R}}^N}\varphi h u_{p}^q\ \textrm{d}x=\int _{{\mathbb {R}}^N}\varphi h u^q\ \textrm{d}x. \end{aligned}$$
(4.24)

We have

$$\begin{aligned} h(x)u_p^q(x) \rightarrow h(x) u^q(x),\ \text{ a.e. }\ x \in {\mathbb {R}}^N. \end{aligned}$$
(4.25)

On the other hand, by \((H_2)\), it yields

$$\begin{aligned} \varphi hu_p^q \le |\varphi |_{\infty } hu_{p}^q =|\varphi |_{\infty } \frac{ h}{b^q}b^q u_{p}^q. \end{aligned}$$

Having in mind that \(b^qu_{p}^q\rightharpoonup b^q u^q\) weakly in the Lebesgue space \(L^\frac{1}{q}({\mathbb {R}}^N)\) and that \(h /b^q \in L^\frac{1}{1-q}({\mathbb {R}}^N)\) which is the dual of \(L^\frac{1}{q}({\mathbb {R}}^N)\), we can assert that \(hu_{p}^q\rightarrow hu^q\) strongly in \(L^1({\mathbb {R}}^N)\). Consequently, there exists \(g\in L^1({\mathbb {R}}^N)\) such that, up to a subsequence,

$$\begin{aligned} h(x)(u_{p}(x))^q \le g(x),\ \text{ a.e. }\ x \in {\mathbb {R}}^N,\ \forall 1< p < 2. \end{aligned}$$
(4.26)

Using (4.25) and (4.26), we can easily apply the Lebesgue-dominated convergence theorem to deduce (4.24). Combining (4.21), (4.22), (4.23), (4.24) and taking the limit in the inequality (4.20) as \(p\rightarrow 1^+\), it follows that

$$\begin{aligned} \int _{{\mathbb {R}}^N}\varphi a|D u| \le \int _{{\mathbb {R}}^N}h u^q \varphi \textrm{d}x -\int _{{\mathbb {R}}^N}u\textrm{az} \cdot \nabla \varphi \textrm{d}x- \int _{{\mathbb {R}}^N}u b \gamma \varphi \textrm{d}x,\ \forall \ \varphi \in C_c^{\infty }( {\mathbb {R}}^N),\ \varphi \ge 0.\nonumber \\ \end{aligned}$$
(4.27)

Now, let us show that

$$\begin{aligned} - u^*\ \text{ div }\left( a z\right) +u\ b\gamma \ge h u^{q},\ \text{ in }\ D'({\mathbb {R}}^N). \end{aligned}$$

Let \(\varphi \in C^{\infty }_{c}({\mathbb {R}}^N)\) be such that \( \varphi \ge 0 \) and \((\rho _n)_n\) a standard sequence of mollifiers. Clearly, \((u*\rho _n)\varphi \in C_c^{\infty }( {\mathbb {R}}^N) \) and \( (u * \rho _n) \varphi \ge 0. \) Then, we can take it as test function (4.18) getting

$$\begin{aligned} -\int _{{\mathbb {R}}^N} (u*\rho _n)\varphi \ \text{ div }\left( a z\right) \ge \int _{{\mathbb {R}}^N}\frac{h}{u^{1-q}} (u*\rho _n)\varphi \textrm{d}x-\int _{{\mathbb {R}}^N}b \gamma (u*\rho _n)\varphi \textrm{d}x.\nonumber \\ \end{aligned}$$
(4.28)

We know that

$$\begin{aligned} u*\rho _n (x)\rightarrow u (x),\ \text{ a.e }\ x \in {\mathbb {R}}^N,\ \text{ as }\ n \rightarrow + \infty , \end{aligned}$$

and

$$\begin{aligned} u* \rho _n(x) \rightarrow u^*(x),\ {\mathcal {H}}^{N-1}-\text{ a.e. }\ \ x \in {\mathbb {R}}^N(\text{ and } \text{ by } \text{ consequence }\ \text{ div }(\textrm{az})-\text{ a.e. }\ \ x \in {\mathbb {R}}^N), \end{aligned}$$

Having in mind that \(|u*\rho _n (x)|\le |u|_{ L^{\infty }({\mathbb {R}}^N)},\ \forall \ x \in {\mathbb {R}}^N,\ \forall \ n \ge 1, \) and \(b \gamma \varphi \in L^1( {\mathbb {R}}^N),\ \frac{h}{u^{1-q}}\varphi \in L^1({\mathbb {R}}^N), \) we can pass to the limit in both sides of the inequality (4.28) using the Lebesgue’s dominated convergence theorem with respect to the measure \(\text{ div }(a z)\) (more precisely with respect to the regular measure that locally represents \( \text{ div }(a z \)) on the left-hand side and to the Lebesgue measure on the right-hand side. Hence,

$$\begin{aligned} -\int _{{\mathbb {R}}^N} u^* \varphi \ \text{ div }\left( a z\right) \ge \int _{{\mathbb {R}}^N}h u^{q}\varphi \textrm{d}x-\int _{{\mathbb {R}}^N}b \gamma u \varphi \textrm{d}x,\ \forall \varphi \in C^{\infty }_{c}({\mathbb {R}}^N),\ \varphi \ge 0. \end{aligned}$$

Inserting that inequality in (4.27), we obtain

$$\begin{aligned} \int _{{\mathbb {R}}^N}\varphi a|D u| \le -\int _{{\mathbb {R}}^N}\varphi u^*\ \text{ div }(\textrm{az}) -\int _{{\mathbb {R}}^N}u\textrm{az} \cdot \nabla \varphi \textrm{d}x= \left\langle \left( \textrm{az},Du\right) , \ \varphi \right\rangle , \qquad \end{aligned}$$
(4.29)

for all \( \varphi \in C_c^{\infty }( {\mathbb {R}}^N),\ \varphi \ge 0. \) Then,

$$\begin{aligned} a|D u| \le \left( \textrm{az},\mathrm{{Du}}\right) , \end{aligned}$$

as measures in \({\mathbb {R}}^N, \) which implies that in fact \( a|D u| = \left( \textrm{az},\mathrm{{Du}}\right) ,\) since the opposite inequality follows from Corollary 1. \(\square \)

To conclude the proof of Theorem 1.1, it remains to show that \(\chi _{\{u> 0 \}}\) belongs to \( \textrm{BV}_{\textrm{loc}}({\mathbb {R}}^N)\) and

$$\begin{aligned} -\left( \text{ div }\big (a z\big )\right) \chi ^*_{\{u>0\}} +b\gamma \chi _{\{u>0\}}=\frac{h}{u^{1-q}} \chi _{\{u>0\}}\ \text{ in }\ D'({\mathbb {R}}^N). \end{aligned}$$
(4.30)

Lemma 9

u solves the equation (4.30).

Proof

For \(\delta >0,\) we define the function \( S_{\delta } : {\mathbb {R}} \rightarrow {\mathbb {R}} \) by

$$\begin{aligned} S_\delta (s)={\left\{ \begin{array}{ll} 0, &{} s\le \delta , \\ \frac{s}{\delta } -1, &{} \delta< s< 2\delta , \\ 1, &{} s\ge 2 \delta . \end{array}\right. } \end{aligned}$$

Let \( \varphi \in C_c^{\infty }( {\mathbb {R}}^N) \) be such that \( \varphi \ge 0. \) Taking \(S_\delta (u_p)\varphi \) as test function in (3.1), we obtain

$$\begin{aligned} \begin{aligned}&\int _{{\mathbb {R}}^N}a|\nabla u_p|^{p} S'_\delta (u_p) \varphi \textrm{d}x+\int _{{\mathbb {R}}^N}a\left| \nabla u_p\right| ^{p-2}\nabla u_p. \nabla \varphi S_\delta (u_p)\textrm{d}x\\&+\int _{{\mathbb {R}}^N}bu_p^{p-1} S_\delta (u_p)\varphi \textrm{d}x=\int _{{\mathbb {R}}^N}\frac{h}{u_p^{1-q}} S_\delta (u_p)\varphi \textrm{d}x. \end{aligned} \end{aligned}$$
(4.31)

Moreover, from (4.31), using Young’s inequality, we have

$$\begin{aligned} \begin{aligned} \int _{{\mathbb {R}}^N}a|\nabla S_\delta (u_p)|\varphi \ \textrm{d}x&+\int _{{\mathbb {R}}^N}a|\nabla u_{p}|^{p-2}\nabla u_{p}.\nabla \varphi S_\delta (u_p)\ \textrm{d}x +\int _{{\mathbb {R}}^N}b u_{p}^{p-1} S_\delta (u_p) \varphi \ \textrm{d}x\\&=\int _{{\mathbb {R}}^N}a|\nabla u_p| S'_\delta (u_p)\varphi \ \textrm{d}x+\int _{{\mathbb {R}}^N}a|\nabla u_{p}|^{p-2}\nabla u_{p}.\nabla \varphi S_\delta (u_p) \ \textrm{d}x\\&\quad +\int _{{\mathbb {R}}^N}bu_{p}^{p-1} S_\delta (u_p) \varphi \ \textrm{d}x\\&\le \frac{1}{p} \int _{{\mathbb {R}}^N}a|\nabla u_{p}|^pS'_\delta (u_p)\varphi \ \textrm{d}x+\frac{p-1}{p}\int _{{\mathbb {R}}^N}aS'_\delta (u_p)\varphi \ \textrm{d}x\\&\quad +\int _{{\mathbb {R}}^N}a|\nabla u_{p}|^{p-2}\nabla u_{p}.\nabla \varphi S_\delta (u_p) \ \textrm{d}x+\int _{{\mathbb {R}}^N}bu_{p}^{p-1} S_\delta (u_p)\varphi \ \textrm{d}x\\&\le \int _{{\mathbb {R}}^N}\frac{h}{u_p^{1-q}}S_\delta (u_p) \varphi \ \textrm{d}x +\frac{p-1}{p}\int _{{\mathbb {R}}^N}aS'_\delta (u_p)\varphi \ \textrm{d}x. \end{aligned} \end{aligned}$$
(4.32)

We have

$$\begin{aligned} |S_\delta (u_p)|\le 1,\ \text{ and }\ |\nabla S_\delta (u_p)|=|S'_\delta (u_p) \nabla u_p|\le \frac{1}{\delta }|\nabla u_p|. \end{aligned}$$

Since \((u_p)_{1< p < 2}\) is bounded in \( W^{1,1}_{a,b} ( {\mathbb {R}}^N) \) (and by consequence in \( \textrm{BV}^b_a({\mathbb {R}}^N)), \) then \( (S_\delta (u_p))_{1< p < 2}\) is a bounded family in \(\textrm{BV}^b_a({\mathbb {R}}^N). \) It follows that

$$\begin{aligned} \int _{{\mathbb {R}}^N} \varphi a |D (S_\delta (u))|\le \liminf _{p \rightarrow 1^+}\int _{{\mathbb {R}}^N} \varphi a |\nabla S_\delta (u_p)| \textrm{d}x. \end{aligned}$$
(4.33)

Moreover, we have

$$\begin{aligned} \frac{h}{u_p^{1-q}}S_{\delta }(u_p)\varphi \le \frac{h}{u_p^{1-q}}\chi _{\{u_p > 2\delta \}}\varphi \le (2\delta )^{q-1}h\varphi \in L^1({\mathbb {R}}^N). \end{aligned}$$

Taking into account that \(\frac{h}{u_p^{1-q}}S_{\delta }(u_p)\varphi \rightarrow \frac{h}{u^{1-q}}S_{\delta }(u)\varphi \) a.e. in \( {\mathbb {R}}^ N,\) we can use the Lebesgue’s dominated convergence theorem to obtain

$$\begin{aligned} \int _{{\mathbb {R}}^N}\frac{h}{u_p^{1-q}}S_\delta (u_p) \varphi \ \textrm{d}x\rightarrow \int _{{\mathbb {R}}^N}\frac{h}{u^{1-q}}S_\delta (u) \varphi \ \textrm{d}x. \end{aligned}$$
(4.34)

Finally, by (4.7) and (4.8), we get

$$\begin{aligned} \int _{{\mathbb {R}}^N}a|\nabla u_{p}|^{p-2}\nabla u_{p}.\nabla \varphi S_\delta (u_p) \ \textrm{d}x\rightarrow \int _{{\mathbb {R}}^N}a\ z \cdot \nabla \varphi S_\delta (u)\ \textrm{d}x \end{aligned}$$
(4.35)
$$\begin{aligned} \int _{{\mathbb {R}}^N}b u_{p}^{p-1} S_\delta (u_p) \varphi \ \textrm{d}x\rightarrow \int _{{\mathbb {R}}^N}S_\delta (u)b\gamma \varphi \textrm{d}x. \end{aligned}$$
(4.36)

Then, from (4.32), (4.33), (4.34), (4.35) and (4.36), it follows that

$$\begin{aligned} \begin{aligned}&\int _{{\mathbb {R}}^N} \varphi a|D S_\delta (u)| +\int _{{\mathbb {R}}^N}a z \cdot \nabla \varphi S_\delta (u) \textrm{d}x + \int _{{\mathbb {R}}^N}S_\delta (u) b \gamma \varphi \textrm{d}x\\&\quad \le \int _{{\mathbb {R}}^N}\frac{h}{u^{1-q}}S_\delta (u)\varphi \textrm{d}x. \end{aligned} \end{aligned}$$
(4.37)

Thanks to the facts that \(z\in {\mathcal {M}}^{\infty }_{a, \textrm{loc}}({\mathbb {R}}^N), \gamma \in L^{\infty }({\mathbb {R}}^N)\) and \(\frac{h}{u^{1-q}}\in L^1_{\textrm{loc}}({\mathbb {R}}^N),\) we can see that all terms except the first one in the inequality (4.37) are uniformly bounded with respect to \(\delta .\) Hence, \( (S_\delta (u))_{\delta > 0}\) is bounded in \(\textrm{BV}_\textrm{loc}({\mathbb {R}}^N)\) and we are allowed to pass to the limit when \(\delta \) tends to 0 (using once again weak lower semicontinuity in the first term and Lebesgue’s theorem for the remaining terms), getting

$$\begin{aligned} \int _{{\mathbb {R}}^N} \varphi a|D\chi _{\{u> 0 \}}| +\int _{{\mathbb {R}}^N}a z \cdot \nabla \varphi \chi _{\{u> 0 \}} \textrm{d}x + \int _{{\mathbb {R}}^N} b \gamma \chi _{\{u> 0 \}} \varphi \textrm{d}x \le \int _{{\mathbb {R}}^N}\frac{h}{u^{1-q}}\chi _{\{u> 0 \}}\varphi \textrm{d}x. \end{aligned}$$

It yields

$$\begin{aligned} \chi _{\{u> 0 \}}\in \textrm{BV}_{\textrm{loc}}({\mathbb {R}}^N), \end{aligned}$$
(4.38)

and

$$\begin{aligned} a|D\chi _{\{u> 0 \}}|-\text{ div }\left( \chi _{\{u> 0 \}}\textrm{az}\right) + b \gamma \chi _{\{u> 0 \}} \le \frac{h}{u^{1-q}}\chi _{\{u> 0 \}},\ \text{ in }\ D'({\mathbb {R}}^N). \end{aligned}$$
(4.39)

From Corollary 1, we infer

$$\begin{aligned} (\textrm{az},D\chi _{\{u>0\}})\le a|D\chi _{\{u>0\}}|. \end{aligned}$$
(4.40)

Also, by Proposition 3, we have \(a z\chi _{\{u>0\}}\in {\mathcal {D}}{\mathcal {M}}_{a, \textrm{loc}}^{\infty }({\mathbb {R}}^N)\), and the following equality holds

$$\begin{aligned} -\text{ div }\left( \textrm{a z}\right) \chi ^*_{\{u>0\}}= \left( a z,D \chi _{\{u>0\}}\right) -\text{ div }\left( a z\ \chi _{\{u>0\}}\right) ,\ \text{ in }\ D'({\mathbb {R}}^N). \end{aligned}$$
(4.41)

Combining (4.41), (4.40) and (4.39), we deduce that

$$\begin{aligned} - \text{ div }\left( \textrm{a z}\right) \chi ^*_{\{u>0\}} +b \gamma \chi _{\{u> 0 \}} \le \frac{h}{u^{1-q}}\chi _{\{u>0\}}. \end{aligned}$$
(4.42)

Next, we try to prove the reverse inequality. For that aim, we take \((\chi _{\{u>0\}}*\rho _n)\varphi \) as test function in (4.18) where \(0\le \varphi \in C^{\infty }_c({\mathbb {R}}^N)\) and \((\rho _n)_n\) is a sequence of mollifiers. By the virtue of the Lebesgue’s dominated convergence theorem and Fatou’s lemma, one can pass to the limit as n tends to \( + \infty \) obtaining

$$\begin{aligned} -\int _{{\mathbb {R}}^N}\varphi \chi ^*_{\{u>0\}} \text{ div }\left( \textrm{a z}\right) + \int _{{\mathbb {R}}^N} b \gamma \chi _{\{u> 0 \}} \varphi \textrm{d}x \ge \int _{{\mathbb {R}}^N} \frac{h}{u^{1-q}} \chi _{\{u> 0 \}} \varphi \textrm{d}x. \end{aligned}$$

It yields

$$\begin{aligned} -\text{ div }\left( \textrm{a z}\right) \chi ^*_{\{u>0\}} + b \gamma \chi _{\{u> 0 \}} \ge \frac{h}{u^{1-q}}\chi _{\{u>0\}}. \end{aligned}$$
(4.43)

Combining (4.43) and (4.42), deduce that (4.30) holds true. \(\square \)

5 Proof of Theorem (1.2)

Let u be the solution given by Theorem 1.1. We already know that \((a(x)z,\mathrm{{Du}})=a(x)|\mathrm{{Du}}|\) as measures in \( {\mathbb {R}}^N.\) Moreover, in view of Remark 3, \(u(x)>0\) a.e. \( x \in {\mathbb {R}}^N. \) Consequently, \( \gamma (x) = 1, \) a.e. \( x \in {\mathbb {R}}^N, \) and

$$\begin{aligned} -\text{ div }(a z) + b \gamma =\frac{h}{u^{1-q}},\ \text{ in }\ D'({\mathbb {R}}^N). \end{aligned}$$

Plainly, it suffices to prove that \(\frac{h}{u^{1-q}} \in L^1({\mathbb {R}}^N).\) First, observe that

$$\begin{aligned} -\int _{{\mathbb {R}}^N} \varphi \ \text{ div }\left( \textrm{az} \right) + \int _{{\mathbb {R}}^N} b \varphi \textrm{d}x= \int _{{\mathbb {R}}^N}\frac{h}{u^{1-q}} \varphi \textrm{d}x,\ \forall \ \varphi \in C_c^{\infty }( {\mathbb {R}}^N). \end{aligned}$$
(5.1)

Given \(0\le v \in W^{1,1}_{a,b}({\mathbb {R}}^N). \) We claim that

$$\begin{aligned} -\int _{{\mathbb {R}}^N} v\ \text{ div }\left( \textrm{az} \right) + \int _{{\mathbb {R}}^N} b v \textrm{d}x= \int _{{\mathbb {R}}^N}\frac{h}{u^{1-q}} v \textrm{d}x. \end{aligned}$$
(5.2)

According to Proposition 4, there exists a sequence \( (\varphi _k)_k \subset C^{\infty }_c({\mathbb {R}}^N)\) such that \( \varphi _k \ge 0 \) and

$$\begin{aligned} \varphi _k \rightarrow v\ \text{ strongly } \text{ in }\ W^{1,1}_{a,b}({\mathbb {R}}^N). \end{aligned}$$

Consider now a sequence \( (\rho _n)_n \) of mollifiers. Set \( v \wedge \varphi _k:=\inf \left\{ v,\ \varphi _k\right\} . \) Taking \( \rho _n*(v \wedge \varphi _k) \) as test function in (5.1), we get

$$\begin{aligned} -\int _{{\mathbb {R}}^N}( \rho _n*(v \wedge \varphi _k)) \text{ div }\left( \textrm{az}\right) + \int _{{\mathbb {R}}^N} b (\rho _n*(v \wedge \varphi _k) )\textrm{d}x= \int _{{\mathbb {R}}^N}\frac{h}{u^{1-q}}(\rho _n*(v \wedge \varphi _k))\textrm{d}x.\nonumber \\ \end{aligned}$$
(5.3)

Clearly, \( v \wedge \varphi _k \in W_{a,b}^{1,1}( {\mathbb {R}}^N). \) Then, having the boundedness of a and b in mind, it follows that

$$\begin{aligned} \rho _n*(v \wedge \varphi _k) \rightarrow v \wedge \varphi _k,\ \text{ strongly } \text{ in }\ W^{1,1}_{a,b}({\mathbb {R}}^N),\ \text{ as }\ n \rightarrow + \infty . \end{aligned}$$

Hence,

$$\begin{aligned} \int _{{\mathbb {R}}^N}\textrm{az} \cdot \nabla \left( \rho _n*(v \wedge \varphi _k)\right) \textrm{d}x \rightarrow \int _{{\mathbb {R}}^N}\textrm{az} \cdot \nabla \left( v \wedge \varphi _k\right) \textrm{d}x,\ n \rightarrow + \infty , \end{aligned}$$
(5.4)

and

$$\begin{aligned} \int _{{\mathbb {R}}^N}b \left( \rho _n*(v \wedge \varphi _k)\right) \textrm{d}x \rightarrow \int _{{\mathbb {R}}^N}b\left( v \wedge \varphi _k\right) \textrm{d}x,\ n \rightarrow + \infty . \end{aligned}$$
(5.5)

For the right-hand side of (5.3), note that there exists a compact \( K_k \subset {\mathbb {R}}^N \) independent of n such that \( \text{ supp }\left( \rho _n*(v \wedge \varphi _k)\right) \subset K_k,\ \forall \ n \ge 1. \) Moreover, \(\left| \rho _n*(v \wedge \varphi _k)\right| _{L^{\infty }({\mathbb {R}}^N)} \le \left| v \wedge \varphi _k\right| _{L^{\infty }({\mathbb {R}}^N)},\ \forall \ n \ge 1,\) and \( \rho _n*(v \wedge \varphi _k) (x)\) converges a.e. \( x \in \ {\mathbb {R}}^N\) to \( (v \wedge \varphi _k)(x)\) as \( n \rightarrow + \infty . \) Consequently, up to a subsequence, \( \rho _n*(v \wedge \varphi _k)\) is \( \text{ weakly}^* \) (weakly star) convergent to \(v \wedge \varphi _k\) in \(L^{\infty }({\mathbb {R}}^N) \) as \( n \rightarrow + \infty . \) Since \(\frac{h}{u^{1-q}} \in L^1(K_k), \) we deduce that,

$$\begin{aligned} \int _{{\mathbb {R}}^N}\frac{h}{u^{1-q}}(\rho _n*(v \wedge \varphi _k))\textrm{d}x\rightarrow \int _{{\mathbb {R}}^N}\frac{h}{u^{1-q}}(v \wedge \varphi _k)\textrm{d}x,\ \text{ as }\ n \rightarrow + \infty . \end{aligned}$$
(5.6)

Combining (5.4), (5.5) and (5.6), we obtain

$$\begin{aligned} \int _{{\mathbb {R}}^N}\textrm{az} \cdot \nabla \left( v \wedge \varphi _k\right) \textrm{d}x + \int _{{\mathbb {R}}^N}(v \wedge \varphi _k) b \textrm{d}x= \int _{{\mathbb {R}}^N}\frac{h}{u^{1-q}}(v \wedge \varphi _k)\textrm{d}x,\ \forall \ k. \end{aligned}$$
(5.7)

Now, we are going to pass to the limit in (5.7) as \(k \rightarrow +\infty . \) Since \( v \wedge \varphi _k \rightarrow v\ \text{ in }\ W^{1,1}_{a,b}({\mathbb {R}}^N),\) then

$$\begin{aligned} \int _{{\mathbb {R}}^N}\textrm{a z} \cdot \nabla \left( v \wedge \varphi _k\right) \textrm{d}x \rightarrow \int _{{\mathbb {R}}^N}\textrm{a z} \cdot \nabla v\textrm{d}x, \end{aligned}$$
(5.8)

and

$$\begin{aligned} \int _{{\mathbb {R}}^N}b \left( v \wedge \varphi _k\right) \textrm{d}x \rightarrow \int _{{\mathbb {R}}^N}b v\textrm{d}x. \end{aligned}$$
(5.9)

We claim that

$$\begin{aligned} \int _{{\mathbb {R}}^N}\frac{h}{u^{1-q}}(v \wedge \varphi _k)\textrm{d}x\rightarrow \int _{{\mathbb {R}}^N}\frac{h}{u^{1-q}}v\textrm{d}x,\ \text{ as }\ k \rightarrow +\infty . \end{aligned}$$
(5.10)

We have

$$\begin{aligned} \frac{h(x)}{(u(x))^{1-q}}(v \wedge \varphi _k)(x) \rightarrow \frac{h(x)}{(u(x))^{1-q}}v(x),\ \text{ a.e. }\ x \in {\mathbb {R}}^N, \end{aligned}$$

and

$$\begin{aligned} 0\le \frac{h}{u^{1-q}}(v \wedge \varphi _k) \le \frac{h}{u^{1-q}} v. \end{aligned}$$

On the other hand, using the strong convergence of \( (\varphi _k)_k \) to v in \( W^{1,1}_{a,b}( {\mathbb {R}}^N), \) it yields

$$\begin{aligned} \int _{{\mathbb {R}}^N}\frac{h}{u^{1-q}} \varphi _k \textrm{d}x= & {} \int _{{\mathbb {R}}^N}\textrm{az} \cdot \nabla \varphi _k \textrm{d}x + \int _{{\mathbb {R}}^N} b \varphi _k \textrm{d}x \\ {}\le & {} |z|_{L^{\infty }( {\mathbb {R}}^N, {\mathbb {R}}^N)}\int _{{\mathbb {R}}^N} a|\nabla \varphi _k|\textrm{d}x + \int _{{\mathbb {R}}^N} b \varphi _k \textrm{d}x \\ {}\le & {} C,\ \forall \ k, \end{aligned}$$

where C is some positive constant independent of k. Then, an application of the Fatou’s lemma implies that \(\frac{h}{u^{1-q}}v\in L^1({\mathbb {R}}^N).\) Therefore, our claim (5.10) follows from the Lebesgue’s dominated convergence theorem. Combining (5.8), (5.9) and (5.10) and passing to the limit in (5.7) as \( k \rightarrow + \infty , \) we infer that the claim (5.2) holds true. In particular, choosing \(v =1, \) we obtain

$$\begin{aligned} \int _{{\mathbb {R}}^N}\frac{h}{u^{1-q}}\textrm{d}x \le \int _{{\mathbb {R}}^N}b\textrm{d}x< +\infty . \end{aligned}$$

This implies that \(\frac{h}{u^{1-q}}\in L^1({\mathbb {R}}^N).\) By consequence, \( \text{ div }(a z) \in L^1( {\mathbb {R}}^N). \)

Since \( \text{ div }(\textrm{az}) \in L^1( {\mathbb {R}}^N) \) and \( \frac{h}{u^{1-q}} \in L^1( {\mathbb {R}}^N), \) then we can easily see that u satisfies the more large following identity:

$$\begin{aligned} - \int _{{\mathbb {R}}^N} v \text{ div }(\textrm{az}) \textrm{d}x + \int _{{\mathbb {R}}^N} b v \textrm{d}x = \int _{{\mathbb {R}}^N} \frac{h}{u^{1-q}} v \textrm{d}x,\ \forall \ v \in \textrm{BV}_a^b( {\mathbb {R}}^N) \cap L^{\infty }( {\mathbb {R}}^N).\nonumber \\ \end{aligned}$$
(5.11)

In order to complete the proof of Theorem 1.2, it remains to show that u is unique. Assume, for the sake of contradiction, that the problem (1.1) admits another positive solution \( u' \) in the sense of Definition (1.5). Thus, there exists a vector field \( z' \in L^{\infty }( {\mathbb {R}}^N, {\mathbb {R}}^N) \) satisfying all the requirements of Definition (1.5). Taking \( v = (u-u') \) as test function in (5.11) for both solutions u and \( u', \) we obtain

$$\begin{aligned} -\int _{{\mathbb {R}}^N} (u-u') \text{ div }(\textrm{az}) \textrm{d}x + \int _{{\mathbb {R}}^N} b (u-u') \textrm{d}x = \int _{{\mathbb {R}}^N} \frac{h}{u^{1-q}} (u-u') \textrm{d}x, \end{aligned}$$
(5.12)

and

$$\begin{aligned} -\int _{{\mathbb {R}}^N} (u-u') \text{ div }(\textrm{az}') \textrm{d}x + \int _{{\mathbb {R}}^N} b (u-u') \textrm{d}x = \int _{{\mathbb {R}}^N} \frac{h}{(u')^{1-q}} (u-u') \textrm{d}x. \qquad \end{aligned}$$
(5.13)

By the Gauss–Green’s formula given in Theorem 2.2, identities (5.12) and (5.13) become

$$\begin{aligned} \int _{{\mathbb {R}}^N} (\textrm{az},\mathrm{{Du}}) - \int _{{\mathbb {R}}^N} (a(x)z,Du') + \int _{{\mathbb {R}}^N} b (u-u') \textrm{d}x = \int _{{\mathbb {R}}^N} \frac{h}{u^{1-q}} (u-u') \textrm{d}x, \nonumber \\ \end{aligned}$$
(5.14)

and

$$\begin{aligned} -\int _{{\mathbb {R}}^N} (\textrm{az}',\mathrm{{Du}}') + \int _{{\mathbb {R}}^N} (\textrm{az}',\mathrm{{Du}}) + \int _{{\mathbb {R}}^N} b (u-u') \textrm{d}x = \int _{{\mathbb {R}}^N} \frac{h}{(u')^{1-q}} (u-u') \textrm{d}x. \nonumber \\ \end{aligned}$$
(5.15)

Having in mind that \( (a z , \mathrm{{Du}}) = a \left| \mathrm{{Du}}\right| \) and \( (\textrm{az}',\mathrm{{Du}}') = a \left| \mathrm{{Du}}'\right| , \) subtracting (5.15) from (5.14), it follows

$$\begin{aligned} \begin{aligned}&\int _{{\mathbb {R}}^N} a \left| \mathrm{{Du}}\right| - \int _{{\mathbb {R}}^N} (\textrm{az}',\mathrm{{Du}}) + \int _{{\mathbb {R}}^N} a \left| \mathrm{{Du}}'\right| - \int _{{\mathbb {R}}^N} (\textrm{az},\mathrm{{Du}}') \\ {}&\quad = \int _{{\mathbb {R}}^N} \left( \frac{h}{u^{1-q}} - \frac{h}{(u')^{1-q}}\right) (u-u') \textrm{d}x. \end{aligned} \end{aligned}$$
(5.16)

By Corollary 1, we know that

$$\begin{aligned} \int _{{\mathbb {R}}^N} a(x) \left| \mathrm{{Du}}\right| - \int _{{\mathbb {R}}^N} (\textrm{az}',\mathrm{{Du}}) \ge 0,\ \text{ and }\ \int _{{\mathbb {R}}^N} a \left| \mathrm{{Du}}'\right| - \int _{{\mathbb {R}}^N} (\textrm{az},\mathrm{{Du}}') \ge 0, \end{aligned}$$

we deduce from (5.16) that

$$\begin{aligned} \int _{{\mathbb {R}}^N} \left( \frac{h}{u^{1-q}} - \frac{h}{(u')^{1-q}}\right) (u-u') \textrm{d}x = - \int _{{\mathbb {R}}^N} \frac{h}{(uu')^{1-q}} \left( u^{1-q} - (u')^{1-q}\right) (u-u')\textrm{d}x \ge 0. \end{aligned}$$

Since \( \left( u^{1-q} - (u')^{1-q}\right) (u-u') \ge 0, \) we immediately deduce that \( u(x) = u'(x) \) a.e. \( x \in {\mathbb {R}}^N. \) This ends the proof of Theorem 1.2.